commerce&audit

90
CIVIL SERVICES (Pre.) EXAM., 2008 Commerce : Solved Paper (Held on 18-5-2008) 1. In case of any default made in complying with the provisions relating to the postal ballot, the officer on default will be punishable with which one of the following fines ? (A) Rs. twenty thousand in respect of each such default (B) Rs. fifty thousand in respect of each such default (C) Rs. one lakh in respect of each such default (D) Rs. two lakh in respect of each such default 2. A listed company opting for buyback of shares under the Companies Act, 1956 has to submit return, after completion of such buy-back within which one of the following periods ? (A) 6 months of such completion to the Registrar of Companies only (B) 45 days of such completion to the SEBI only (C) 30 days of such completion to the Registrar of Companies and SEBI (D) 30 days of such completion to the SEBI only 3. Which one of the following is correct in respect of the doctrine of indoor management and the rule of constructive notice ? (A) The former operates to protect the outsider against the company and the latter protects the company against the outsider (B) The former operates to protect the company against the outsider and the latter protects the outsider against the company (C) Both protect the company against the outsider (D) Both protect the outsider against the company 4. What is Director Identification Number (DIN) ? (A) An identification number which the individual company allots to the intending director (B) A number which the Central Government allots to any individual intending to be appointed as director or to any existing director of a company (C) A number which the SEBI allots to any individual intending to be appointed as director or to any existing director of a company (D) A number which the Central Government allots to retired directors so as to build data base 5. Small shareholders' director is to be appointed by which of the following companies ? (A) Public companies having Rs. 5 crores or more of paid up capital and minimum of 1000 small shareholders (B) Public companies having Rs. 10 crores or more of paid up capital and a

Upload: taniya-mahtur

Post on 08-Nov-2014

19 views

Category:

Documents


7 download

DESCRIPTION

comeerce

TRANSCRIPT

Page 1: Commerce&Audit

CIVIL SERVICES (Pre.) EXAM., 2008

Commerce : Solved Paper(Held on 18-5-2008)

1. In case of any default made in complying with the provisions relating to the postal ballot, the officer on default will be punishable with which one of the following fines ?(A) Rs. twenty thousand in respect of each such default(B) Rs. fifty thousand in respect of each such default(C) Rs. one lakh in respect of each such default(D) Rs. two lakh in respect of each such default 

2. A listed company opting for buyback of shares under the Companies Act, 1956 has to submit return, after completion of such buy-back within which one of the following periods ?(A) 6 months of such completion to the Registrar of Companies only(B) 45 days of such completion to the SEBI only(C) 30 days of such completion to the Registrar of Companies and SEBI(D) 30 days of such completion to the SEBI only

3. Which one of the following is correct in respect of the doctrine of indoor management and the rule of constructive notice ?(A) The former operates to protect the outsider against the company and the latter protects the company against the outsider (B) The former operates to protect the company against the outsider and the latter protects the outsider against the company(C) Both protect the company against the outsider(D) Both protect the outsider against the company

4. What is Director Identification Number (DIN) ?(A) An identification number which the individual company allots to the intending director (B) A number which the Central Government allots to any individual intending to be appointed as director or to any existing director of a company(C) A number which the SEBI allots to any individual intending to be appointed as director or to any existing director of a company (D) A number which the Central Government allots to retired directors so as to build data base

5. Small shareholders' director is to be appointed by which of the following companies ?(A) Public companies having Rs. 5 crores or more of paid up capital and minimum of 1000 small shareholders(B) Public companies having Rs. 10 crores or more of paid up capital and a minimum of 1000 small shareholders(C) Public companies having Rs. 1 crore or more of paid up capital and a minimum of 100 small shareholders(D) Public companies having Rs. 5 crores or more of paid up capital and a minimum of 100 small shareholders

6. When does a body corporate become capable forthwith of exercising all the functions of a company ?(A) On finalizing Memorandum of Association(B) On obtaining certificate of commencement of business(C) On obtaining certificate of incorporation(D) On having convened its first Annual General Meeting

7. For how many years the books of accounts of a company have to be preserved in good order ?

Page 2: Commerce&Audit

(A) For a period of not less than eight years immediately preceding the current year(B) For a period of not less than five years immediately preceding the current year(C) For a period of not less than six years immediately preceding the current years(D) For a period of not less than four years immediately preceding the current year

8. Which one of the following is not one of the elements of the 7S McKinsey Model for analyzing and improving organizational effectiveness ?(A) Strategy(B) Structure(C) Shared Values(D) Standard

9. Consider the following statements—1. Aggregate planning implies operational planning concerned with determining a firm's production requirements and manufacturing capacity.2. Behaviour modification is a concept of motivation totally independent of rewards for the employees.Which of the statements given above is/are correct ?(A) 1 only(B) 2 only(C) Both 1 and 2(D) Neither 1 nor 2

10. What is the correct sequence of the following steps in a strategic planning process ?1. Inputs2. Evaluation of alternatives3. Execution4. SWOT analysisSelect the correct answer using the code given below—(A) 1 - 4 - 2 - 3 (B) 4 - 1 - 2 - 3(C) 1 - 2 - 3 - 4 (D) 4 - 1 - 3 - 2

11. Consider the following statements—1. Managerial grid is an organizational concept that brings together teams of individuals from specialized jobs and departments for achieving specific results indefined periods of time. 2. The matrix management model is based on a matrix of values between 0 and 9 for two primary variables. Which of the statements given above is/are correct ?(A) 1 only(B) 2 only(C) Both 1 and 2(D) Neither 1 nor 2

12. A limited company makes a net profit of Rs. 2,00,000 after writing off preliminary expenses amounting to Rs. 20,000 and providing for depreciation on assets amounting to Rs. 40,000 and gain of Rs. 10,000 on sale of a piece of machinery. What are the funds from operation ?(A) Rs. 2,00,000(B) Rs. 2,40,000(C) Rs. 2,50,000(D) Rs. 2,60,000

13. Consider the following items that are disclosed in a statement of cash flows—1. Cash flow from operating activities.2. Cash flow from financing activities.

Page 3: Commerce&Audit

3. Closing cash and cash equivalents.4. Cash from investing activities What is the correct chronological order of their disclosure of the above cash flows ?(A) 2 - 3 - 1 - 4 (B) 1 - 4 - 2 - 3(C) 2 - 4 - 1 - 3 (D) 1 - 3 - 2 - 4

14. Consider the following statements—A cash flow statement is generally prepared for1. Ascertaining the net cash flows between two accounting periods.2. Understanding the trends of cash flows between two accounting periods.3. Short term cash planning of the business.Which of the statements given above are correct ?(A) 1 and 2 only(B) 2 and 3 only(C) 1 and 3 only(D) 1, 2 and 3

15. Which of the following represents corporate sustainable reporting ?(A) Publishing annual financial statement only(B) Publishing annual financial statements and social accounts(C) Publishing social and environmental accounts annually(D) Publishing annual financial statements incorporating economic, social and environmental performances

16. Sometimes an auditor is called upon to review the operations of an enterprise for evaluating their cost-effectiveness. What is this kind of audit generally known as ?(A) Cost Audit(B) Operations Audit(C) Tax Audit(D) Independent Financial Audit

17. Match List-I with List-II and select the correct answer using the code given below the Lists—List-I (Assets and Liabilities)(a) Current assets(b) Fixed assets(c) Deferred revenue expenditure(d) Current liabilitiesList II (Accepted Principles of Valuation)1. To be valued at cost less depreciation2. To be valued at cost or market price whichever is lower3. To be valued at the maximum figure which might be incurred4. To be written off within a reasonable period of timeCodes :(a) (b) (c) (d)(A) 2 1 4 3(B) 3 4 1 2(C) 2 4 1 3(D) 3 1 4 2

18. Match List-I with List-II and select the correct answer using the code given below the Lists—List I (Type of Audit)(a) Statutory Audit(b) Management Audit(c) Cost Audit

Page 4: Commerce&Audit

(d) Financial AuditList II (Explanation)1. Judge the correctness of the financial statements and establish their reliability2. Appraisal of performance of executives3. Examination of past records4. Verification of cost recordsCodes :(a) (b) (c) (d)(A) 3 4 2 1(B) 1 2 4 3(C) 3 2 4 1(D) 1 4 2 3

19. Match List-I with List-II and select the correct answer using the code given below the Lists—List-I (Technique of Auditing)(a) Vouching(b) Verification(c) Investigation(d) ValuationList-II (Explanation)1. It is an enquiry into the value, ownership and title of assets2. It is testing of the exact value of an asset on the basis of its utility3. It is the verification of authority and authenticity of transactions as recorded in the books of accounts 4. It is an examination of accounts and records of a business concern with some special purpose Codes :(a) (b) (c) (d)(A) 3 2 4 1(B) 4 1 3 2(C) 3 1 4 2(D) 4 2 3 1

20. Consider the following statements—Internal check relates to1. arrangement of duties of employees.2. distribution of work in such a way that work of one employee is checked by other employee.3. review of operations. Which of the statements given above is/are correct ?(A) 1 and 2 only(B) 2 and 3 only(C) 3 only(D) 1, 2 and 3

21. Consider the following categories of documentary evidence from the point of view of the degree of their reliability.1. Documentary evidence originating from and held by the entity.2. Documentary evidence originating from third parties and held by the entity.3. Documentary evidence originating from and held by the third party.4. Documentary evidence originating from entity and held by the third party.Which one of the following is the correct order (high to low) of their degree of reliability ?(A) 4 - 1 - 3 - 2 (B) 3 - 2 - 4 - 1(C) 4 - 2 - 3 - 1 (D) 3 - 1 - 4 - 2

22. Which one of the following is not included in the internal check as regards purchases ?(A) Assessment of requirements

Page 5: Commerce&Audit

(B) Placing orders(C) Despatch of goods(D) Recording and making payments

23. Consider the following statements—1. It is mandatory for all business enterprises to conduct audit of accounts.2. Audit Note Book is a tool in the conduct of audit.3. Internal check system is an inbuilt mechanism within an organization to check frauds and errors.Which of the statements given above is/are correct ?(A) 1 only(B) 2 and 3 only(C) 3 only(D) 1, 2 and 3

24. Match List-I with List-II and select the correct answer using the code given below the Lists—List-I (Type of Books)(a) Minutes Book(b) Subsidiary Ledger(c) Stores Ledger(d) Log BookList-II (Content)1. Records of issue of packing materials2. Records of running hours of machine3. Records of decisions at a meeting4. Records entry of receipt from debtorsCodes :(a) (b) (c) (d)(A) 3 2 1 4(B) 1 4 3 2(C) 3 4 1 2(D) 1 2 3 4

25. What is the method in which an amount equal to the amount written off as depreciation is invested in outside securities in order to facilitate replacement of an asset at the expiry of its life period, called ?(A) Annuity Method(B) Sinking Fund Method(C) Replacement Method(D) Depletion Method

26. What is Capital Redemption Reserve Account available for ?(A) Redemption of redeemable preference shares(B) Redemption of redeemable debentures(C) Reorganization of share capital(D) Issue of fully paid bonus shares

27. Match List-I with List-II and select the correct answer using the code given below the Lists—List I (Item)(a) Earnings per share(b) Cash flow statement(c) Inventory valuation(d) Disclosure of accounting policiesList-II (Accounting Standard)1. AS-3 2. AS-203. AS-1 4. AS-2

Page 6: Commerce&Audit

Codes :(a) (b) (c) (d)(A) 3 1 4 2(B) 2 4 1 3(C) 3 4 1 2(D) 2 1 4 3

28. Government grants related to income as per the Accounting Standard (AS)-12 should be—(A) Presented as a credit in the statement of Profit and Loss(B) Presented as a deferred income on the asset side of the balance sheet(C) Presented as a capital grant on the liability side of balance sheet(D) Presented both in the Profit and Loss statement and in the balance sheet

29. What is the renewal fees paid for a patent right ?(A) Capital expenditure(B) Revenue expenditure(C) Deferred revenue expenditure(D) Development expenditure

30. According to the Accounting Standard (AS)-2, the inventory is to be valued at which one of the following ?(A) Actual cost or sales value, whichever is lower(B) Historical cost(C) Net realizable value(D) Historical cost or net realizable value, whichever is lower

31. Match List-I with List-II and select the correct answer using the code given below the Lists—List-I (Nature of Expenditures/Receipts)(a) Capital receipts(b) Capital expenditure(c) Revenue expenditure(d) Revenue receiptList II (Example)1. Sale of fixed assets2. Improvement of fixed assets3. Income received as interest4. Interest on loan for businessCodes :(a) (b) (c) (d)(A) 1 2 4 3(B) 3 4 2 1(C) 1 4 2 3(D) 3 2 4 1

32. Which of the following is affected by treatment of an accrued item in accounting ?(A) Profit and Loss A/c only(B) Profit and Loss Appropriation A/c(C) Balance Sheet only(D) Profit and Loss A/c and Balance Sheet

33. Income and expenses for the year 2006 are— Fee Received in Cash — Rs.24,000 Accrued Fee — Rs.6,000Rent for Chamber Paid — Rs.6,000Outstanding Rent — Rs.2,000Salary Paid to Staff — Rs.6,000

Page 7: Commerce&Audit

Miscellaneous Expenses Paid — Rs.200Salary paid in advance to staff included in the above : Rs. 1,000.What is the income under accrual basis ?(A) Rs. 11,800 (B) Rs. 13,200(C) Rs. 16,800 (D) Rs. 30,000

34. If Opening Stock — Rs.15,000Purchases — Rs.37,500Direct Expenses — Rs.1,500Closing Stock — Rs.7,500Operating Expenses — Rs.3,000Sales are Rs. 60,000 during the year, what is the net profit ?(A) Rs. 12,000 (B) Rs. 10,500(C) Rs. 7,500 (D) Rs. 3,000

35. On 1.4.2007, a firm's capital was Rs. 2,00,000, Assets Rs. 2,40,000 and Liabilities Rs. 40,000. It sold goods costing Rs. 10,000 for Rs. 12,000. What will be the position of accounting equation after this transaction has been recorded in accounts ?Assets = Capital + Liabilities(A) 2,42,000 = 2,02,000 + 40,000(B) 2,52,000 = 2,12,000 + 40,000(C) 2,30,000 = 2,00,000 + 30,000(D) 2,22,000 = 1,82,000 + 40,000

36. Which one of the following statements about preparation of financial statements of a limited liability company is correct ?(A) Financial statements shouldbe prepared according to the cash basis of accounting only(B) Financial statements should be prepared according to the accrual basis of accounting only(C) Financial statements should be prepared according to either the cash basis or accrual basis of accounting(D) Financial statements should be prepared according to both the accrual and cash basis of accounting

37. Expenses need to be recorded in the period in which the associated revenues are recognised. This is to be ensured as per which one of the following accounting principles ?(A) Revenue recognition(B) Cost benefit(C) Matching(D) Periodicity

38. How is profit prior to incorporation treated as ?(A) Revenue reserve(B) Secret reserve(C) Capital reserve(D) General reserve

39. Which one of the following has replaced the Budla system in India ?(A) Rolling settlement(B) Trading in equity derivatives(C) Depository system(D) Online trading

Page 8: Commerce&Audit

40. Which one of the following is not correct ?(A) Mutual Fund ensures its participants a professional management of portfolio selection(B) Mutual Fund schemes can be open-ended and/or closeended(C) Balanced Mutual Fund aims at earning current income and capital appreciation(D) Mutual Fund is an investment company and a merchant banker

41. Which one of the following statements is not correct ?(A) Business firms combine to avoid wasteful competition(B) Business combination is possible only among joint stock companies(C) Business firms may combine to take advantage of patent rights of individual firms(D) During economic depression, small firms are absorbed by financially sound large firms

42. What is the theory of social responsibility that focuses on how companies respond to issues, rather than trying to determine their ultimate social responsibility, called ?(A) Corporate social performance(B) Corporate social responsiveness(C) Stewardship principle(D) Enlightened self-interest

43. What is a systematic evaluation of a company's activities in a given area of social interest such as environmental protection, workplace safety or communityinvolvement, called ?(A) Social responsibility(B) Social accounting(C) Social audit(D) Social forecasting

44. What is cross rate ?(A) A rate of exchange derived from central bank(B) A rate of exchange between two currencies, other than those that form a market's principal rates(C) A rate of exchange derived from the quotations for buying currencies(D) A rate of exchange quoted by a dealer in foreign exchange for selling currencies

45. Which one of the following statements is not correct ?(A) Working capital is made available by the banks to the export sector at subsidized rates of interest(B) Market Development Fund provides the necessary financial assistance for market promotion(C) Export Houses/Star Trading houses are granted additional license against exports made during the preceding year(D) Export profits enjoy a variety of tax concessions in India

46. Which one of the following statements is correct in respect of Duty Drawback Scheme ?(A) Relieving export goods of duties paid for components used for manufacturing the export goods(B) Providing cash incentives to exporters to compensate the duty paid by them in foreign countries(C) Compensating importers to the extent of import duty paid by them for importing life-saving drugs(D) Providing a facility for withdrawing the duty paid by exporters if their exports are rejected in foreign countries

47. In foreign trade, what is the price quoted by a supplier which includes all charges incurred up to door delivery of goods to the buyer, called ?(A) Loco price(B) C.I.F. price(C) Franco price(D) Landed price

48. Consider the following statements—

Page 9: Commerce&Audit

1. Letter of credit cannot be opened by mail.2. Contracts of export of goods and services against payment to be secured partly or fully beyond 90 days are treated as deferred payment exports.Which of the statements given above is/are correct ?(A) 1 only(B) 2 only(C) Both 1 and 2(D) Neither 1 nor 2

49. Under which principle, all the rights of an insured are transferred to insurance company after making payment of claim ?(A) Subrogation(B) Utmost good faith(C) Contribution(D) Average clause

50. In a marine insurance, when must the insurable interest exist ? (A) At the time of making contract(B) At the time of loss of subject matter(C) Both at the time of making contract and at the time of loss of subject matter(D) At the time of termination of the policy

51. What is/are the consequence(s) of non-registration of partnership firm ?1. It cannot sue any of its partners.2. Partners of an unregistered firm can file suits against each other.3. Partners can sue the firm to enforce their claims.Select the correct answer using the code given below—(A) 1(B) 2(C) 3(D) None of the above

52. Consider the following statements—In the absence of an agreement, a partner of a firm has the right to1. claim remuneration for taking part in the management.2. take part in the conduct and management of business.3. share the profits of the firm equally with other partners.4. pledge partnership property as a security for his own private debts.Which of the statements given above is/are correct ?(A) 1 and 4 (B) 2 and 3(C) 1 only (D) 4 only

53. Match List-I with List-II and select the correct answer using the code given below the Lists—List-I (Type of Company)(a) Statutory company(b) Registered company(c) Company limited by shares(d) Company limited by guaranteeList-II (Feature)1. Member's liability is limited to the face value of shares2. Regulated by the provisions of the Companies Act3. Generally formed for promotion of culture, art, science, religion etc.4. Formed under Special ActCodes :

Page 10: Commerce&Audit

(a) (b) (c) (d)(A) 4 1 2 3(B) 3 2 1 4(C) 3 1 2 4(D) 4 2 1 3

54. Consider the following statements in respect of the “Duty Free Import Authorization Scheme”.1. The scheme offers the facility to import the required inputs before exports.2. The scheme allows the transfer of scrip once the export obligation is complete. Which of the statements given above is/are correct ?(A) 1 only(B) 2 only(C) Both 1 and 2(D) Neither 1 nor 2

55. Under which one of the following is the term ‘Dominant Undertaking’ defined ?(A) MRTP Act(B) FEMA(C) Companies Act(D) SEBI

Directions—(Q. 56–60) Each of the next five items consists of two statements, one labelled as the ‘Assertion’ (A) and the other as ‘Reason’ (R). You are to examine these two statements carefully and select the answers to these items using the codes given below—Codes :(A) Both (A) and (R) are individually true and (R) is the correct explanation of (A).(B) Both (A) and (R) are individually true but (R) is not the correct explanation of (A).(C) (A) is true but (R) is false.(D) (A) is false but (R) is true.

56. Assertion (A)—A company is accountable to the society since it makes use of community-owned assets and other infrastructural facilities and enjoys concessions provided by State from taxpayers' money.Reason (R)—The best way of social responsibility reporting is to prepare a value-added statement which shows the income earned by all the stake-holders.

57. Assertion (A)—Income tax paid is not shown on the debit side of the sole proprietor's Profit and Loss account.Reason (R)—It is an appropriation of profit and thus goes to Profit and Loss Appropriation Account.

58. Assertion (A)—The scope of audit of accounts of a limited company may be restricted by the Board of Directors.Reason (R)—The Board of Directors is responsible for adherence to Accounting Standards in preparation of financial statements.

59. Assertion (A)—Analytical review procedures are useful only at the time of planning of an audit. Reason (R)—Analytical reviews are the test of financial information conducted through a study and review of relationship among financial data.

60. Assertion (A)—In dividend stripping or bond washing which is resorted to in some countries, fixed-interest securities are bought when they have gone exdividend and sold before the next dividend is paid.Reason (R)—In such countries, dividends are taxed at a higher rate than capital gains. 

61. To which one of the following aspects does an established pattern of relationships among the components of an organization refer ?(A) Organization strategy

Page 11: Commerce&Audit

(B) Organization structure(C) Organization process(D) Organization policy

62. Consider the following statements—1. Authority in an organization is the formal or institutional right to command stemming from official delegation of position.2. Legitimate power is the right to manage derived from delegation based on ownership or property rights.Which of the statements given above is/are correct ?(A) 1 only(B) 2 only(C) Both 1 and 2(D) Neither 1 nor 2

63. Which one of the following is not correct in respect of the Theory X? (A) It emphasizes that most average human beings avoid to be led, want to lead(B) According to the theory X, the Managers tend to be autocratic(C) The theory X assumes that most human beings must be directly controlled in order to achieve organizational goals(D) The theory X assumes that most individuals have little ambition and want security 

64. When Maslow's need hierarchy is compared with Herzberg's two-factor theory of motivation, which of the following statements are correct ?1. Maslow's psychological needs; security, safety and affiliation or acceptance represent almost the same meanings as Herzberg's maintenance factor.2. Esteem or status is also a part of maintenance factor.3. Self-actualization can be compared as a component of motivators as in Herzberg's theory.Select the correct answer using the code given below—(A) 1, 2 and 3(B) 1 and 2 only(C) 2 and 3 only(D) 1 and 3 only

65. Which one of the following is not an element of direction ?(A) Planning and organizing various resources for doing work(B) Continuous training activity in which subordinates are instructed to carry out a particular assignment(C) Motivation of subordinates to try to meet the expectations of manager(D) Maintaining discipline and rewarding those who perform properly

66. Consider the following statements in respect of PERT—1. PERT is a variation of milestone budgeting.2. PERT employs probabilistic estimate of time required for completion of an activity. 3. PERT also includes the transfer of resources for reduction of duration of completion of a project.Which of the statements given above are correct ?(A) 1, 2 and 3(B) 1 and 2 only(C) 2 and 3 only(D) 1 and 3 only

67. Some organizations have escalator clause in their labour agreements. What does this clause provide for ?(A) Automatic increase in wage/salary depending on seniority(B) Wage increase depending on output per day by the worker(C) Automatic increase in wage/salary depending upon increase in price index(D) Automatic increase in wage/salary depending on increase in profits of the firm

Page 12: Commerce&Audit

68. What is an organization structure in which each employee reports to both a functional (or division) manager and to a project (or group) manager, known as ?(A) Strategic business unit (B) Departmentation by customer(C) Matrix structure(D) Departmentation by territory

69. According to force-field theory, every behaviour is the result of equilibrium between which of the following ?(A) Push and pull effect(B) Strong and weak forces(C) Positive and negative forces(D) Driving and restraining forces

70. Life insurance is a contract of which one of the following ?(A) Indemnity(B) Guarantee(C) Contribution(D) Subrogation

71. What is outsourcing of production and concentrating on marketing operations in international business, known as ?(A) Licensing(B) Franchising(C) Contract manufacturing(D) Joint venture

72. Which of the following is not a feature of bonds ?(A) Par value(B) Yield to maturity(C) Coupon rate(D) Maturity

73. On 1st April, 2006, Y Ltd. Issued 1000, 12% debentures of Rs. 100 each at a discount of 6%. These debentures are redeemable in five equal annual instalments at the end of each year. What is the amount of discount to be written off in the first year i.e. on 31st March, 2007, through P & L A/c ?(A) Rs. 2000 (B) Rs. 1800(C) Rs. 1200 (D) Rs. 600

74. XYZ Ltd. issued 10,000 shares of Rs. 100 each at Rs. 120 per share with Rs. 25 on application, Rs. 45 on allotment including premium, Rs. 20 on first call and Rs. 30 on final call. Govind who held 200 shares did not pay any call money and his shares were forfeited. What is the amount to be credited to Share Forfeiture Account on forfeiture ?(A) Rs. 14,000 (B) Rs. 10,000(C) Rs. 9,000 (D) Rs. 4,000

75. X Ltd. purchased a car from Maruti Udyog Ltd. for Rs. 5,00,000. As per agreement Rs. 80,000 was to be paid in cash and the balance by issue of shares of Rs. 10 each at a premium of Rs. 5 per share. How many shares should X Ltd. issue to Maruti Udyog Ltd. for the car ?(A) 30,000 shares(B) 29,000 shares(C) 28,500 shares

Page 13: Commerce&Audit

(D) 28,000 shares

76. A company has issued a 20-year bond at Rs. 1000 par value with a coupon rate of 10 per cent. The required rate of return on similar bonds is 12 per cent. What is the current value of the bond ?(A) Rs. 905•33 (B) Rs. 850•45(C) Rs. 833•33 (D) Rs. 820•20

77. Consider the following statements—Current ratio is increased by1. issue of redeemable preference shares.2. selling of old furniture for cash.3. cash realized from debtors.Which of the statements given above are correct ?(A) 1 and 2 only(B) 2 and 3 only(C) 1 and 3 only(D) 1, 2 and 3

78. Which one of the following statements is correct ?When creditors' velocity or creditors' turnover is higher as compared to debtors' velocity, it would(A) improve liquidity(B) reduce liquidity(C) have no effect on liquidity(D) improve financial position

79. Sundry Debtors — Rs.15,000Bills Receivable (B/R) — Rs.12,500Cash at Bank — Rs.17,500Stock — Rs.15,000Profit — Rs.20,000Creditors — Rs.25,000Bills Payable (B/P) — Rs.15,000Sales — Rs.1,00,000What is the Acid Test Ratio ?(A) 1•5 : 1 (B) 1•125 : 1(C) 16 2/3% (D) 50%

80. Which one of the following transactions changes the current ratio ?(A) Purchase of goods for cash(B) Plant acquired on account(C) Sold goods on credit(D) Debentures converted into equity capital

81. Few items of P/L A/c of a company are— Sales — Rs.1,60,000Closing stock — Rs.38,000Non-operating Expenses — Rs.800Non-operating Income — Rs.4,800Net Profit — Rs.28,000What is the Operating Profit Ratio ?(A) 18% 

Page 14: Commerce&Audit

(B) 20%(C) 15% (D) 57%

82. Net profit after tax of Rs. 2,00,000 is Rs. 4,00,000. Share capital is Rs. 8,00,000 and revenue reserve is Rs. 2,00,000. What is rate of return on equity ?(A) 40% (B) 50%(C) 60% (D) 75%

83. Members of a company may apply to which one among the following for relief under the Companies Act, 1956 in cases of oppression ?(A) Central Government(B) High Court of Judicature(C) National Company Law Tribunal(D) National Company Law Appellate Tribunal

84. A company limited by shares has to call the statutory meeting within a period of not less than one month and not more than six months. This period is counted with reference to which one of the following ?(A) The date of incorporation(B) The date at which the company is entitled to commence business(C) The date of actual receipt of certificate of incorporation(D) The date of actual commencement of business

85. Which among the following is eligible to issue Shelf Prospectus ?(A) Any company contemplating for public issue of securities(B) Any public financial institution(C) Only manufacturing companies(D) Only foreign companies engaged in trading in India

86. Which of the following companies do not have the obligation to get its Articles of Association registered along with the Memorandum of Association ?(A) Public Company limited by shares(B) Unlimited companies(C) Private companies limited by shares(D) Companies limited by guarantee 

87. What does management audit imply ?(A) Complete audit(B) Detailed audit(C) Efficiency audit(D) Interim audit

88. Which one of the following statements is correct ?(A) Internal audit and Management audit are the same(B) Internal audit and statutory audit are the same(C) Internal audit is compulsory in all cases(D) Statutory audit of company accounts is compulsory

89. Where does an auditor of a cooperative society submit the audit report ?(A) To the managing committee of the society only(B) To the Registrar of Cooperative Societies of the State concerned only(C) To the State Assembly concerned (D) To the Registrar of Cooperative Societies of the State concerned and a copy to the society

Page 15: Commerce&Audit

90. Which one of the following statements is correct ?(A) Audit of an educational institution is compulsory if it is run by a charitable trust(B) A club is treated as a commercial establishment(C) The accounts of a charitable trust can be audited by any person who belongs to accountancy profession(D) Audit of a charitable trust is not compulsory under law

91. Consider the following statements—The vouching of remuneration to directors involve the checking of1. special resolution.2. statement of accounts.3. approval note of the Central Government.4. Minutes Book.Which of the statements given above are correct ?(A) 1 and 2 only(B) 2 and 3 only(C) 1, 2 and 4(D) 1, 3 and 4

92. Match List-I with List-II and select the correct answer using the code given below the Lists—List-I (Liability of Auditor)(a) Liability for negligence(b) Liability for misfeasance(c) Criminal liability(d) Liability to third partyList-II (Example)1. Breach of trust or duty2. Untrue statement in the prospectus3. Not liable for untrue statement in prospectus4. Dividends paid out of capitalCodes :(a) (b) (c) (d)(A) 4 3 2 1(B) 2 1 4 3(C) 4 1 2 3(D) 2 3 4 1

93. Consider the following statements—1. Corporate governance requires the appointment of audit committees by publiclimited companies. 2. The members of an audit committee should be those non-executive directors who do not represent the controlling group.3. The independence of the statutory auditors is likely to be affected if he deals directly with the audit committee. Which of the statements given above are correct ?(A) 1 and 2 only(B) 2 and 3 only(C) 1 and 3 only(D) 1, 2 and 3

94. Under the head ‘Secured Loans’ the following are disclosed—1. Loans and Advances from banks.2. Debentures.3. Loans and Advances from subsidiaries.4. Other loans and advances.What is the correct chronological order of disclosure of the items given above in balance sheet of a

Page 16: Commerce&Audit

company ?(A) 3 - 1 - 2 - 4 (B) 2 - 4 - 3 - 1(C) 3 - 4 - 2 - 1 (D) 2 - 1 - 3 - 4

95. Consider the following statements—Accounting control comprises1. The plan of organization concerned with decision process.2. The plan of organization concerned with safeguarding of assets.3. The procedure concerned with safeguarding of assets. Which of the statements given above are correct ?(A) 1 and 2 only(B) 2 and 3 only(C) 1 and 3 only(D) 1, 2 and 3

96. Which of the following statements is/are correct ?1. Internal control includes quality control.2. Internal control is compulsory for all forms of business organizations.3. Internal control helps external auditor to design suitable audit programme for client's organization.Select the correct answer using the code given below—(A) 1 only (B) 2 only(C) 3 only (D) 1 and 3

97. Which of the following is not a part of the usual contents of the current file and audit working papers ?(A) A copy of accounts on which auditors are reporting(B) Audit programme(C) Minutes of the meetings(D) Memorandum of Association and Articles of Association

98. Consider the following statements—Audit working papers are maintained because1. they act as a guide to subsequent audit.2. they verify the arithmetical accuracy of the books of accounts.3. they provide evidence of the work performed by the auditor.Which of the statements given above are correct ?(A) 1 and 2 only(B) 2 and 3 only(C) 1 and 3 only(D) 1, 2 and 3

99. Match List-I with List-II and select the correct answer using the code given below the Lists—List-I (Technique)(a) Rotation of Duties(b) Random Sampling(c) Trend Analysis(d) Cash verificationList-II (Auditing Procedure)(1) Internal control(2) Physical examination(3) Selective verification(4) Overall assessmentCodes :

Page 17: Commerce&Audit

(a) (b) (c) (d)(A) 1 4 3 2(B) 2 3 4 1(C) 1 3 4 2(D) 2 4 3 1

100. Consider the following stages an auditor has to undergo before commencing an audit—1. Ascertaining the scope of audit work.2. Gaining knowledge about business of the client.3. Going through the agreement with the client.4. Instructions to and information from the client.What is the correct sequence of the above stages ?(A) 2 - 1 - 3 - 4(B) 3 - 4 - 2 - 1(C) 2 - 4 - 3 - 1(D) 3 - 1 - 2 – 4

101. Which one of the following statements is correct ?(A) If an asset is used by a company, it should be owned by the company(B) Verification of liabilities means that the amounts of current and long-term liabilities shown in the balance sheet are correct(C) Vouching and verification means the same thing (D) It is not the duty of an auditor to confirm the physical existence and ownership of an asset

102. The following steps should be adopted by an auditor for verification of land—1. To examine that the land has been shown in the balance sheet at its original cost.2. To ensure that the cost of land also includes the expenses like broker's commission, legal expenses, improvement expenses etc.3. To get a certificate from solicitor of the client regarding validity of the title deeds.4. To examine that the land is not subject to any charge or mortgage.What is the correct sequence of the above steps to be followed by an auditor during the course of auditing ?(A) 1 - 4 - 2 - 3(B) 3 - 2 - 4 - 1(C) 1 - 2 - 4 - 3(D) 3 - 4 - 2 – 1

103. What does payout ratio mean ?(A) Ratio of debtors to creditors(B) Ratio of profit distributed to profit retained(C) Ratio of dividends per share and earnings per share(D) Retained earnings

104. Consider the following statements—1. Premium on issue of shares is transferred to General Reserve Account.2. For declaration of bonus shares out of General Reserve, a resolution in the shareholders' meeting is necessary.Which of the statements given above is/are correct ?(A) 1 only(B) 2 only(C) Both 1 and 2(D) Neither 1 nor 2

105. How is ‘Calls in arrear’ shown in the balance sheet on the liability side ?(A) By deducting the amount from the called up capital(B) Under the heading ‘Reserves and Surplus’

Page 18: Commerce&Audit

(C) By deducting the amount from ‘Capital Reserve’(D) By deducting the amount from the ‘Share Forfeited Account’

106. Match List-I with List-II and select the correct answer using the code given below the Lists—List-I (Activity)(a) Admission of a partner(b) Partnership dissolution(c) Rectification of errors(d) Transfer of realization profitList-II (Account)(1) Capital Accounts(2) Suspense Account(3) Revaluation Account(4) Realization AccountCodes :(a) (b) (c) (d)(A) 3 4 2 1(B) 2 1 3 4(C) 3 1 2 4(D) 2 4 3 1

107. The balance of old provision for doubtful debts on 1-4-2006 was Rs. 10,000. The bad debts written off during the year 2006-07 amounted to Rs. 12,000, and the new provision requiredon 31-3-2007 was Rs. 15,000. What is the total amount to be debited to profit and loss account on account of bad debts and the provision for doubtfuldebts ?(A) Rs. 37,000(B) Rs. 27,000(C) Rs. 17,000(D) Rs. 15,000

108. A and B sharing profit in the ratio of 3 : 2 and having capitals of Rs. 30,000 (for A) and Rs. 15,000 (for B), decided to dissolve their firm. After paying off all liabilities, cash realized from various assets is Rs. 15,000. How will this amount be distributed to A and B ?(A) A–Rs. 9,000 and B–Rs. 6,000(B) A–Rs. 10,000 and B–Rs. 5,000(C) A–Rs. 7,500 and B–Rs. 7,500(D) A–Rs. 12,000 and B–Rs. 3,000

109. X and Y sharing profits in the ratio of 7 : 3 admit Z on 37th share in the new firm. Z takes 27 th from X and 17th from Y.What is the new ratio among X, Y and Z ?(A) 7 : 3 : 3 (B) 4 : 2 : 15(C) 14 : 6 : 15 (D) 29 : 11 : 30

110. Given—Opening inventory : Rs. 3,500Closing inventory : Rs. 1,500Cost of goods sold : Rs. 22,000What is the amount of purchase ?(A) Rs. 20,000(B) Rs. 24,000(C) Rs. 27,000(D) Rs. 17,000

Page 19: Commerce&Audit

111. Consider the following—Amortization refers to writing off the value of(1) tangible assets(2) intangible assets(3) fictitious assetsWhich of the above is/are correct ?(A) 1 and 2 (B) 2 and 3(C) 1 only (D) 2 only

112. The following information is disclosed by ‘A’— Rs. Provision for doubtful debts as on 1.1.2006 : 6,000 Bad debts written off during the year 2006 : 1,200 Total debtors as on31.12.2006 : 80,000 A provision for Doubtful Debts to be made @ 5% What is the amount to be shown on the credit side of P/L A/c ?(A) Rs. 4,800 (B) Rs. 2,400(C) Rs. 1,200 (D) Rs. 800

113. What is reduction of paid-up share capital called ?(A) External reconstruction(B) Internal reconstruction(C) Reorganization(D) Redemption of capital

114. Under which of the following methods of depreciation the amount of an asset is never reduced to zero ?(A) Straight line method(B) Diminishing balance method(C) Sums of Years Digit method(D) Annuity method

115. An insurance claim of Rs. 300 was accepted in respect of stock (inventory) of Rs. 500, which was destroyed by fire. Rs. 200 not covered by insurance should be debited to which one of the following ?(A) Stock account(B) Trading account(C) Profit and loss account(D) Goodwill account

116. Debtors turnover ratio is 4. What is the average collection period ?(A) 5 months (B) 4 months(C) 3 months (D) 2 months

117. The net profits of a business after providing for taxation for the past five years are Rs. 80,000, Rs. 85,000, Rs. 92,000, Rs. 1,05,000 and Rs. 1,18,000. Capital employed in the business is Rs. 8,00,000. Normal rate of return is 10%. What is the value of goodwill on the basis of capitalization of super profit method ?(A) Rs. 1,00,000(B) Rs. 1,50,000(C) Rs. 1,60,000(D) Rs. 1,80,000

Page 20: Commerce&Audit

118. Given— Purchases — Rs.50,000Sales — Rs.90,000Closing Stock — Rs.7,000Manufacturing Exp. — Rs.5,000Rate of gross profit — Rs.3313% on costWhat is the opening stock ?(A) Rs. 20,000(B) Rs. 19,500(C) Rs. 22,500(D) Rs. 25,000

119. A particular firm provided the following data for a year—Current Ratio 2•5 : 1Liquid Ratio 1•5 : 1Net Working CapitalRs. 3,00,000What are the current assets and current liabilities of this firm, respectively ?(A) Rs. 3,00,000 and Rs. 1,50,000(B) Rs. 5,00,000 and Rs. 2,00,000(C) Rs. 2,00,000 and Rs. 5,00,000(D) Rs. 3,00,000 and Rs. 1,00,000

120. Depreciation is a process of which one of the following ?(A) Valuation of assets(B) Allocation of acquisition cost over the estimated useful life of the asset(C) Allocation of realizable value over the estimated useful life of the asset(D) Estimating the market value of the asset on the balance sheet date

Answers with Hints1. (B) 2. (C) 3. (A) 4. (B)5. (A) Small shareholders of a public company will be able to appoint (elect) one director on the Board of Company if the company has a paid up capital of Rs. 5 crore or more, or has 1000 or more small shareholders as per section 252 Companies (Amendment) Act, 2000.6. (C) 7. (A)8. (D) 7S McKinsey Model's elements includes :(i) Strategy, (ii) Structure, (iii) System, (iv) Shared value, (v) Skills, (vi) Style, (vii) Staff9. (C) 10. (D) 11. (C)12. (C) Funds from operation computed as follows :Net profit — Rs.2,00,000Add : Preliminary Exp. — Rs.20,000Depreciation — Rs.40,000= 2,60,000Less : Gain on sale of machinery — Rs.10,000Funds from operation = 2,50,00013. (C) 14. (C) 15. (D) 16. (A) 17. (A) 18. (C) 19. (C)20. (A) According to Spicer and Pegler internal check is an arrangement of staff duties whereby no one person is allowed to carry through and record every aspect of a transaction so that, without collusion between any two or more persons, fraud is prevented and at the same time the possibilities of errors are

Page 21: Commerce&Audit

reduced to a minimum.21. (B) 22. (C) 23. (C) 24. (C) 25. (B) 26. (D) 27. (D) 28. (D) 29. (B) 30. (D)31. (A) 32. (D) 33. (B) 34. (B) 35. (B) 36. (B) 37. (A) 38. (C) 39. (A) 40. (D) 41. (B) 42. (A) 43. (C) 44. (B) 45. (C) 46. (C) 47. (C) 48. (C) 49. (A) 50. (B) 51. (D) 52. (B) 53. (D) 54. (C) 55. (C) 56. (A) 57. (D) 58. (D) 59. (A) 60. (B) 61. (B) 62. (C) 63. (A) 64. (B) 65. (C) 66. (C) 67. (D)68. (C) An organisation in which each employee reports to both a functional manager and to a project manager as Matrix Structure of organisation.69. (D) 70. (B) 71. (B) 72. (B) 73. (C) 74. (B) 75. (D) 76. (A) 77. (A) 78. (C)79. (B) 80. (B) 81. (C) 82. (A) 83. (C) 84. (B) 85. (B) 86. (A) 87. (C) 88. (D) 89. (B) 90. (A) 91. (D) 92. (C) 93. (A) 94. (D) 95. (B) 96. (D) 97. (D) 98. (C) 99. (C) 100. (D) 101. (B) 102. (B) 103. (C) 104. (B) 105. (A) 106. (A) 107. (C) 108. (A) 109. (D) 110. (A) 111. (D) 112. (D) 113. (B) 114. (B) 115. (C) 116. (C) 117. (C) 118. (B) 119. (B) 120. (B)

R.A.S./R.T.S. (Pre.) Examination, 2008(Held on 7-1-2009)

Commerce : Solved Paper

1. The minimum paid-up share capital for a public company shall be—(A) Rs. 1 lakh(B) Rs. 2 lakhs(C) Rs. 3 lakhs(D) Rs. 5 lakhsAns : (D)

2. Internal activity of a company is going to be performed according to established regulations. This assumption is provided as a right by—(A) Doctrine of Indoor Management(B) Doctrine of Constructive Notice(C) Doctrine of Ultravires(D) Doctrine of IntraviresAns : (A)

3. A public company can start its business operations after getting—(A) Certificate of Incorporation(B) Minimum Subscription(C) Certificate of Commencement of Business(D) Permission of the Controller of Capital IssueAns : (C)

4. Under Section 275 of the Companies’ Act, 1956 a person can become director in public company of not more than—(A) 5 companies(B) 10 companies(C) 15 companies(D) 20 companiesAns : (C)

5. Articles of Association can be altered by passing—(A) An ordinary resolution in Annual General Meeting

Page 22: Commerce&Audit

(B) A special resolution in Annual General Meeting(C) A resolution with special notice(D) Without any resolutionAns : (B)

6. In case of Board Meetings the Quorum must be present—(A) At the commencement of the meeting(B) At the termination of the meeting(C) Throughout the meeting(D) At the commencement and termination bothAns : (A)

7. In a public company the minimum number for having a Quorum in a meeting is—(A) 2(B) 3(C) 5(D) 7Ans : (C)

8. The capital issues of public limited companies are subject to guidelines issued by—(A) Reserve Bank of India(B) Central Government(C) Central Bank of India(D) Securities & Exchange Board of India (SEBI)Ans : (D)

9. Disinvestment of shares means—(A) To sale the shares of private company to public(B) To sale the shares of public company to the public(C) To sale the shares of Government company to the public(D) To sale of shares by holding company to its subsidiary companyAns : (B)

10. When the existing companies raise additional funds by issue of shares to the existing shareholders in proportion to their existing shareholdings, it is called—(A) Buyback of shares(B) Issue of shares at premium(C) Issue of shares at discount(D) Right shares issueAns : (D)

11. Bombay Stock Exchange Sensex consists of ……… script as on 31st March, 2005.(A) 10(B) 20(C) 30(D) 40Ans : (C)

12. The basic aim of Securities and Exchange Board of India (SEBI) is to—(A) Develop an effective and efficient monitoring and control system for the Indian Capital Market(B) To help Bombay Stock Exchange only(C) To help National Stock Exchange of India Ltd. only(D) To help OTC Exchange of India onlyAns : (A)

Page 23: Commerce&Audit

13. Who is Father of Scientific Management ?(A) Henry Fayol(B) Elton Mayo(C) Chester Bernard(D) F. W. TaylorAns : (D)

14. Principle of ‘Unity of Command’ means—(A) Unity of directions(B) To receive orders from one officer only(C) A fixed place for every person(D) Proper delegation to subordinatesAns : (B)

15. Managerial Planning is a—(A) Middle Level Function(B) Lower Level Function(C) Top Level Function(D) All Level FunctionAns : (D)

16. “Organisation is the foundation of management.” This statement is given by—(A) Henry Ford(B) Haney(C) Keeling(D) Lansberry FishAns : (A)

17. The number of subordinates a superior can effectively handle is called—(A) Cooperation(B) Coordination(C) Supervision(D) Span of controlAns : (D)

18. In Need Hierarchy Theory of Abraham H. Maslow, Self Actualisation needs are at level—(A) Lower level needs(B) Middle level needs(C) Higher level needs(D) Highest level needsAns : (A)

19. Motivation - Hygiene Theory was propounded by—(A) Abraham H. Maslow(B) Fredrick Herzberg(C) Peter F. Druker(D) Argyris ChrisAns : (B)

20. ‘Carrot and Stick’ principle is given in Theory—(A) McGregor Theory X(B) McGregor Theory Y(C) William Ouchi Theory Z(D) None of the aboveAns : (C)

Page 24: Commerce&Audit

21. “Organisation is a Management Process by which people, functions and physical factors are brought together to form a controllable unit.” This definition is given by—(A) Haney(B) Kelling B. Lewis(C) Oliver Sheldon(D) CornellAns : (C)

22. Out of the following forms, which form of organisation is the oldest one ?(A) Line and staff organisation(B) Functional organisation(C) Committee organisation(D) Line organisationAns : (D)

23. “A body of persons elected or appointed to meet on an organised basis for the discussion and dealing of matters brought before it.” It is called—(A) Functional organisation(B) Formal organisation(C) Committee organisation(D) Informal organisationAns : (C)

24. Due to coordination—(A) There is a unity of order(B) There is a unity of direction(C) There is a leadership(D) Employees are motivatedAns : (A)

25. In ‘Direction’ who is given importance ?(A) To machines(B) To paper work(C) To man(D) To productionAns : (C)

26. The Essence of Control is—(A) To take decision(B) To take corrective action(C) To make enquiry(D) To motivateAns : (B)

27. An annual general meeting may be called after giving shorter notice instead of 21 clear days, if consent is accorded by—(A) All the directors of company(B) All the members entitled to vote there at(C) Majority of members entitled to vote there all(D) None of the aboveAns : (B)

28. “The position of a Company Secretary is like that of a hub in a bicycle wheel.” This statement shows the—(A) Importance of Company Secretary(B) Rights of Company Secretary

Page 25: Commerce&Audit

(C) Duties of Company Secretary(D) Liabilities of Company SecretaryAns : (A)

29. The first Secretary of the company is appointed by—(A) Shareholders(B) Promoters(C) Government(D) DirectorsAns : (B)

30. Appointment of full time Company Secretary is compulsory in all those companies, whose paidup share capital is—(A) Rs. 50 lakhs or more(B) Rs. 1 crore or more(C) Rs. 2 crores or more(D) Rs. 3 crores of moreAns : (C)

31. A meeting of the shareholders held only once during the life time of the company is known as—(A) Meeting of the Directors(B) Meeting of the Creditors(C) Extraordinary General Meeting(D) Statutory MeetingAns : (D)

32. For calling the Extraordinary General Meeting a clear notice of ……… days must be given to all the members.(A) 7(B) 15(C) 21(D) 30Ans : (C)

33. Appointment of a person who is a Director, as a Secretary in the Company would require approval of the company by—(A) Special Resolution(B) Ordinary Resolution(C) Resolution by circulation(D) None of the aboveAns : (A)

34. The first Stock Exchange in India was established in—(A) Calcutta(B) Delhi(C) Bombay(D) MadrasAns : (C)

35. Blue Chips Shares mean—(A) Those shares which are listed in Stock Exchange(B) Those shares whose guarantee is given by Government(C) Those shares on whom dividend is paid at higher rate regularly(D) Those shares which are issued at first timeAns : (C)

Page 26: Commerce&Audit

36. FEMA stands for—(A) Foreign Exchange Management Act(B) Funds Exchange Management Act(C) Finance Enhancement Monetary Act(D) Future Exchange Management ActAns : (A)

37. What of the following is false about W.T.O. ?(A) It is the main organ for implementing the Multilateral Trade Agreement(B) It is global in its membership(C) It has far wider scope than GATT(D) Only countries having more than prescribed level of total GDP can become its memberAns : (D)

38. Convertibility of the rupee implies—(A) Being able to convert rupees notes into gold(B) Allowing the value of the rupee to be fixed by marketforces(C) Freely permitting the conversion of rupee to other major currencies and vice versa(D) Developing an international market for currencies in IndiaAns : (C)

39. ……… has been founded to act as permanent watchdog on the international trade.(A) ISRD(B) ADS(C) WTO(D) DIMFAns : (C)

40. Which of the following statement is correct ?(A) The disinvestment programme has been successfully carried out in India(B) Privatisation up to 100% has been carried out in all the PSU in India(C) Under strategic sale method of disinvestment, the government sells a major share to a strategic partner(D) None of the aboveAns : (C)

41. The present World Economic Depression first of all, started from which country ?(A) U.S.A.(B) U.K.(C) France(D) IndiaAns : (A)

42. Which, institution is known as the ‘Soft Loan Window’ of World Bank ?(A) I.F.C. (International Financial Corporation)(B) I.D.A. (International Development Association)(C) I.M.F. (International Monetary Fund)(D) Indian Development ForumAns : (B)

43. Which is not a insurable risk ?(A) Accident Risk(B) Loss of Crops Risk(C) The Risk of Trading in New Market(D) The Risk of Sinking of a ShipAns : (C)

Page 27: Commerce&Audit

44. The Life Insurance in India was nationalised in the year—(A) 1870(B) 1956(C) 1960(D) 1966Ans : (B)

45. Fire Insurance is based on the principle of—(A) Utmost Good faith(B) Insurable Interest(C) Indemnity(D) CooperationAns : (C)

46. In order to reduce the risk of heavy insurance the insurer passes on some business to the other company, it is called—(A) Reinsurance(B) Double Insurance(C) Joint Insurance Policy(D) Separate InsuranceAns : (A)

47. Contents of Marine Insurance include—(A) Insurance of Cargo only(B) Insurance of Freight only(C) Insurance of Hull only(D) Insurance of Cargo, Freight & HullAns : (D)

48. The object of ‘Agenda’ is to inform—(A) About the profitability and activity(B) About the progress of company(C) About the matter in sequence to be discussed in the meeting(D) About the routine mattersAns : (C)

49. The Branch of Accounting which is concerned with the processing and presenting data for decision making is known as—(A) Common Dollar Accounting(B) Social Accounting(C) Cost Accounting(D) Management AccountingAns : (D)

50. Final accounts prepared in narrative style are in—(A) Horizontal form(B) Accounts form(C) Vertical form(D) None of the aboveAns : (C)

51. According to Balance Sheet equation concept, the capital will be—(A) Capital = Liabilities – Assets(B) Capital = Fixed Assets –Current Assests

Page 28: Commerce&Audit

(C) Capital = Assets – Liabilities(D) Capital = Assets + LiabilitiesAns : (C)

52. In common size Balance Sheet analysis we evaluate—(A) Increase or decrease over two years is analysed(B) Only increase over two years is analysed(C) Only decrease over two years is analysed(D) All assets and liabilities are expressed in terms of percentage of totalAns : (D)

53. Increase in fixed asset due to purchase is—(A) Source of fund(B) Fund from operation(C) Use of fund(D) None of the aboveAns : (C)

54. For the purpose of preparation of fund flow statement, fund means—(A) Total resources(B) Cash/bank balances(C) Current Assets(D) Working capitalAns : (D)

55. The following data, relates to manufacturing company for the year 2006-07—Net Profit as per P & L A/c—Rs. 2,40,000; Depreciation—Rs. 80,000; Goodwill written-off—Rs. 40,000, Profit on Sale of Fixed Assets—Rs. 16,000, Proposed Dividend—Rs. 96,000. The fund from operation would be—(A) Rs. 4,40,000(B) Rs. 4,00,000(C) Rs. 6,40,000(D) None of the aboveAns : (D)

56. Total sales is Rs. 7,60,000, cash sales Rs. 30,000 collection period is 25 days, debtors at Balance Sheet date will be—(A) Rs. 80,000(B) Rs. 70,000(C) Rs. 60,000(D) Rs. 50,000Ans : (D)

57. Price earning ratio is 83•33% and E.P.S. is Rs. 30. The market price of equity share will be—(A) Rs. 33•33(B) Rs. 66•67(C) Rs. 20(D) Rs. 25Ans : (D)

58. If the current ratio is 2, current assests are worth Rs. 1,600, if current ratio is not allowed to fall below 1•5, how much additional can be borrowed by the company on the short term basis ?(A) Rs. 400(B) Rs. 600(C) Rs. 2,733(D) Rs. 800

Page 29: Commerce&Audit

Ans : (A)

59. Rate of Gross Profit on cost is 25%. Total sales is Rs. 1,00,000 and Average Stock is Rs. 1,60,000. Stock Turnover Ratio will be—(A) 0•5 times(B) 0•8 times(C) 0•10 times(D) 0•4 timesAns : (A)

60. Average stock of firm is Rs. 80,000, the opening stock is Rs. 10,000 less than closing stock. Find opening stock.(A) Rs. 95,000(B) Rs. 85,000(C) Rs. 90,000(D) Rs. 75,000Ans : (D)

61. Current Ratio is 3•75, Acid Test Ratio is 1•25 Stock Rs. 3,75,000, calculate working capital.(A) Rs. 3,00,000(B) Rs. 4,00,000(C) Rs. 4,12,500(D) Rs. 4,25,000Ans : (C)

62. From the information given below, calculate Debt service coverage Ratio—Net profit after interest and Tax Rs. 40,000, Depreciation Rs. 5,000, Rate of Income Tax 50%, 10% Mortgage Debentures Rs. 60,000. Fixed Interest Charges Rs. 6,000, Debenture Redemption Fund Appropriation of Outstanding Debentures 10%.(A) 4•06 times(B) 5•06 times(C) 6•06 times(D) 7•06 timesAns : (A)

63. Share premium account can be used for—(A) Paying tax liability(B) Meeting the cost of issue of shares or debentures(C) Paying Dividend on shares(D) Meeting the loss on sale of old assetAns : (B)

64. Ploughing-back of profits means—(A) Dividend declared but not claimed by shareholders(B) Non-declaration of dividend in any year(C) Profits earned from illegal sources and employed in business(D) Retaining the earnings of business for future expansion programmeAns : (D)

65. As per Schedule VI of the Companies’ Act, 1956, Forfeited Share Account will be—(A) Added to paid up capital(B) Deducted from called up capital(C) Added to capital reserve(D) Shown as a revenue reserveAns : (B)

Page 30: Commerce&Audit

66. A company invited application for subscription of 5,000 shares. The application were received for 6,000 shares. The shares were allotted on pro-rata basis. If X has applied for 180 shares how many shares would be allotted to him ?(A) 180 shares(B) 200 shares(C) 150 shares(D) 175 sharesAns : (C)

67. Consider the following information pertaining to K Ltd. on September 4, 2005, the company issued 12,000 7% debentures having a face value of Rs. 100 each at a discount of 2•5%. On September 12, the company issued 25,000 preference shares of Rs. 100 each. On September 29, the company redeemed 30,000 preference shares of Rs. 100 each at a premium of 5% together with one month dividend @ 6% p.a. thereon. Bank balance on August 31, 2005 was Rs. 30,00,000. After effecting the above transaction, the Bank Balance as on September 30, 2005 will be—(A) Rs. 35,15,000(B) Rs. 33,80,000(C) Rs. 33,45,000(D) Rs. 35,05,000Ans : (A)

68. The balance appearing in the books of a company at the end of year were CRR A/c Rs. 50,000, Security Premium Rs. 5,000, Revaluation Reserve Rs. 20,000, P & L A/c (Dr) Rs. 10,000. Maximum amount available for distribution of Bonus Share will be—(A) Rs. 50,000(B) Rs. 55,000(C) Rs. 45,000(D) Rs. 57,000Ans : (B)

69. R. G. Ltd. purchased machinery from K.G. Company for a book value of Rs. 4,00,000. The consideration was paid by issue of 10%. Debenture of Rs. 100 each at a discount of 20%. The debenture account will be credited by—(A) Rs. 4,00,000(B) Rs. 5,00,000(C) Rs. 3,20,000(D) Rs. 4,80,000Ans : (A)

70. Depletion method of depreciation is used in case of—(A) Cattle, Loose Tools, etc.(B) Mines, Quarries, etc.(C) Machinery, Building, etc.(D) BooksAns : (B)

71. V. V. Bros. purchased a machine on 1st October, 2003 at cost Rs. 2,70,000 and spent Rs. 30,000 on its installation. The firm written off depreciation 10% per annum on original cost every year. The books are closed on 31st March every year. The machine is sold on 30 September, 2006 for Rs. 1,90,000. How much amount will be transferred to P & L A/c as loss on sale of machinery ?(A) Rs. 10,000(B) Rs. 20,000(C) Rs. 30,000(D) Rs. 40,000Ans : (B)

Page 31: Commerce&Audit

72. Accounting for Intangible Assets are related to—(A) AS - 10(B) AS - 12(C) AS - 24(D) AS - 26Ans : (D)

73. Indian Accounting Standard – 28 is related to—(A) Accounting for taxes on income(B) Financial Reporting of Interests in Joint Venture(C) Impairment of Assets(D) Provisions, Contingent Liabilities and Contingent AssetsAns : (C)

74. Recording of capital contributed by the owner as liability ensures the adherence of principle of—(A) Consistency(B) Going concern(C) Separate entity(D) MaterialityAns : (C)

75. A company made the purchases of an item during its financial year.January 2007 200 @ Rs. 50 eachMay 2007 400 @ Rs. 60 eachAugust 2007 600 @ Rs. 70 eachNovem- 2007 300 @ Rs. 80 each berClosing inventories were 500 articles. Find out the value of closing stock as per Weighted Average Method—(A) 33,333•33(B) 16,666•66(C) 66,666•67(D) 96,666•66Ans : (A)

76. A fire occurred in the premises of ‘M’ Ltd. on 30th September, 2007. The stock was destroyed except to the extent of Rs. 10,000. From the information given below, calculate the value of stock burnt by fire on 30th September, 2007. Stock on 1st April 2006, Rs. 90,000, Purchases less returns during 2006-07—10,00,000, Sales less returns during 2006-07, 15,00,000; Stock on 31st March, 2007—1,80,000, Purchases less returns from 1st April 2007 to 30th September 2007, 7,00,000. Sales less returns from 1st April, 2007 to 30th September 2007, 10,00,000.It was the practice of the company to value stock less 10%—(A) 3,00,000(B) 2,90,000(C) 1,90,000(D) 2,80,000Ans : (B)

77. Social Accounting means—(A) Accounting for social benefits and social costs(B) Accounting for Government Revenue & Govt. Cost(C) Accounting for private revenue and private cost(D) None of the aboveAns : (A)

78. It is given that cost of stock is Rs. 100. However, its market price is Rs. 98 (buying) and Rs. 140 (selling). If the market price is interpreted as the replacement cost, then the stock should be valued at—

Page 32: Commerce&Audit

(A) Rs. 98(B) Rs. 100(C) Rs. 140(D) Rs. 40Ans : (A)

79. If the goods purchased are in transit, then the Journal Entry at the end of the period will be—(A) Goods in Transit A/c Dr. To Supplier’s A/c(B) Goods in transit A/c Dr. To Purchases A/c(C) Stock A/c Dr. To Goods in Transit A/c(D) Supplier’s A/c Dr. To Goods in TransitAns : (B)

80. Calculate Return on Investment/ Return on Proprietor’s fund. Gross profit of a firm is Rs. 3,20,000, Operating expenses Rs. 1,00,000, Taxes Rs. 20,000, Owner’s fund Rs. 5,00,000, Debenture Interest Rs. 50,000—(A) 20%(B) 30%(C) 40%(D) 50%Ans : (B)

81. The following figures are presented to you—Year Sales Profit/Loss1999 Rs. 1,00,000 Rs. 10,000 (Loss)2000 Rs. 2,50,000 Rs. 20,000 (Profit)Calculate Profit Volume Ratio.(A) 5%(B) 10%(C) 15%(D) 20%Ans : (D)

82. The following particulars relate to manufacturing factory for the month of March 2008. Variable cost per unit Rs. 11; Fixed factory overhead Rs. 5,40,000; Fixed selling overhead Rs. 2,52,000; Variable selling cost Rs. 3; Sales Price per unit Rs. 20. Find out the Break-Even Point in rupees—(A) Rs. 26,40,000(B) Rs. 30,80,000(C) Rs. 25,20,000(D) Rs. 27,40,000Ans : (C)

83. The following information is obtained from the records of K Co. Ltd.—Sales (1,00,000 units) Rs. 1,00,000Variable cost Rs. 40,000Fixed cost Rs. 30,000Find out margin of safety.(A) Rs. 20,000(B) Rs. 25,000(C) Rs. 30,000(D) Rs. 50,000Ans : (D)

84. A manufacturer is operating at 50% of its capacity, due to competition. The following are the details. Raw materials Rs. 6 per unit, Direct Labour Rs. 4 per unit. variable overhead - Rs. 3 per unit, fixed overhead - Rs. 2 per unit, output 15,000 units, total cost Rs. 2,25,000, sales value Rs. 2,10,000, loss Rs.

Page 33: Commerce&Audit

15,000. A foreign customer wants to buy 6,000 units at Rs. 13•50 per unit and the company does not know whether to accept or not as it is suffering losses at the current level. Advise what he should do ?(A) Accept the offer(B) Reject the offer(C) Remains indifferent(D) None of the aboveAns : (B)

85. I took the order for 5,000 units at Rs. 50 each because I got more than the cost incurred to produce them, said the Works Manager, and produced the following figures—

Particulars Before accepting the order After accepting the orderRs. Rs.

Variable costs 2,50,000 4,00,000Fixed costs 7,50,000 8,51,000Total costs 10,00,000 12,51,000Cost/unit 40 41•70Analyse the above figures and the decision taken.Acceptance of this offer has—(A) Lowered his profit by Rs. 500(B) Lowered his profit by Rs. 1,000(C) Lowered his profit by Rs. 1,500(D) Lowered his profit by Rs. 2,000Ans : (C)

86. The following data are provided to you.Fixed cost - Rs. 20,000; Selling price per unit - Rs. 25, Variable cost per unit - Rs. 20. Find out selling price per unit if B.E.P. is brought down to 2,000 units.(A) Rs. 30(B) Rs. 40(C) Rs. 50(D) Rs. 60Ans : (C)

87. Among the following which is not an error of commission ?(A) Wrong totalling(B) Recording with wrong amount(C) Wrong posting(D) Escape from postingAns : (D)

88. Internal check means—(A) Checking of accounts by cashier(B) Checking of accounts by the Internal Auditor(C) Checking the work of one person by another automatically(D) Managerial control internally over the subordinatesAns : (C)

89. Verification includes—(A) Valuation

Page 34: Commerce&Audit

(B) Existence(C) Ownership & Title(D) All of the aboveAns : (D)

90. By whom from the following, auditor can be reappointed ?(A) Directors(B) Shareholders(C) Central Government(D) All of the aboveAns : (D)

91. Auditor shall be punished with imprisonment for a maximum period of ……… under Section 539 for falsification in the books of accounts.(A) 3 years(B) 5 years(C) 7 years(D) 9 yearsAns : (C)

92. “Auditor is not an insurer.” In which of the following cases, the decision has been given ?(A) The Kingston Cotton Mills Co. Ltd. (1986)(B) London & General Bank (1895)(C) Allen Craig & Co. Ltd. (1934)(D) Irish Woollen Co. Ltd.Ans : (B)

93. The Section 80A of the Companies’ Act is related with the redemption of—(A) Debentures(B) Redeemable preference shares(C) Irredeemable preference shares(D) None of the aboveAns : (C)

94. Company Auditor is responsible—(A) For directors(B) For shareholders(C) For public(D) For creditorsAns : (B)

95. In Balance Sheet, Audit Accounts are audited—(A) Monthly(B) Bi-monthly(C) Annually or half yearly(D) QuarterlyAns : (C)

96. Audit adopted by banking company is—

Page 35: Commerce&Audit

(A) Continuous Audit(B) Periodical Audit(C) Internal Audit(D) Balance Sheet AuditAns : (A)

97. Which of the following have lien of Company Auditor ?(A) Books of Accounts of the Company(B) Vouchers of the Company(C) Auditor’s working paper(D) All of the aboveAns : (C)

98. To appoint new auditor in place of retiring auditor, to adopt the procedure, which section of Companies’ Act, 1956 is applicable ?(A) 223(B) 224(C) 225(D) 226Ans : (B)

99. “A company has a separate legal existence from its members.” This principle was first laid down in case of—(A) Saloman Vs. Saloman & Co. Ltd. (1897)(B) Daimler Co. Ltd. Vs. Continental Tyre & Rubber Co. (1916)(C) State of U. P. Vs. Renu Sagar Power Co. (1991)(D) Santunu Roy Vs. Union of India (1989)Ans : (A)

100. According to ownership, Government Company means ……… share.(A) Whose paid-up share capital’s 48%, shares are owned by Government(B) Whose paid-up share capital’s 49% shares are owned by Government(C) Whose paid-up share capital’s 50% shares are owned by Government(D) Whose paid-up share capital’s 51% shares are owned by GovernmentAns : (D)

Uttarakhand PCS (Pre) Exam, 2006 : CommerceUttarakhand P.C.S. (Pre.) Exam., 2006

Commerce : Solved Paper(Held on 9-3-2008)

1. The share of new partner in the profits is 1/5 and his capital is Rs. 20,000. The new profit sharing ratio is 3 : 1 : 1. The share of partners in total capital will be—(A) 60,000 : 20,000 : 20,000(B) 80,000 : 20,000 : 20,000(C) 50,000 : 20,000 : 25,000(D) None of the above

2. At the time of dissolution the loss of the business, will be compensated first of all from—

Page 36: Commerce&Audit

(A) Capital(B) Profits(C) Personal resources of the partners(D) Donations

3. The meaning of written down value is—(A) Original cost – Scrap value(B) Book value + Depreciation(C) Book value – Depreciation(D) None of these

4. Given :Depreciation on the basis of Fixed Instalment Method Rs. 2,000 p.a.Establishment expenses Rs. 5,000Scrap value Rs. 1,000Span of life 10 yearsThe cost of assets will be—(A) Rs. 20,000(B) Rs. 16,000(C) Rs. 12,000(D) None of these

5. Given :Cost Rs. 1,00,000Scrap Value Rs. 10,000Span of Life 10 yearsRate of depreciation 20% p.a.The amount of depreciation for the first year on the basis of diminishing balance method will be—(A) Rs. 20,000 (B) Rs. 18,000(C) Rs. 9,000 (D) Rs. 10,000

6. Match the following List-I with List-II and select the correct answer from the answer codes given below—List-I(a) AS-6 (b) AS-10(c) AS-26 (d) AS-20List-II1. Accounting for Earning per share2. Accounting for intangible and fictitious assets3. Accounting for fixed assets4. Depreciation AccountingCodes :(a) (b) (c) (d)(A) 1 2 3 4(B) 3 2 1 4(C) 1 3 2 4(D) 4 3 2 1

7. Establishment expenses of a new machine will be debited to—(A) Expenses Account(B) Profit and Loss Account(C) Machinery Account

Page 37: Commerce&Audit

(D) None of the above

8. Goodwill is—(A) Floating Asset(B) Wasting Asset(C) Fictitious Asset(D) Intangible Asset

9. A Balance Sheet shows only—(A) Personal Accounts and Nominal Accounts(B) Real Accounts and Nominal Accounts(C) Personal Accounts and Real Accounts(D) Personal, Real and Nominal Accounts

10. Discount on Issue of Shares A/c is shown at the—(A) Debit side of P & L A/c(B) Assets side of B/S(C) Liabilities side of B/S(D) None of the above

11. Pre-incorporation Profit is transferred to—(A) General Reserve(B) Capital Reserve(C) Profit and Loss A/c(D) Trading A/c

12. Which of the following is shown in Profit & Loss Appropriation A/c ?(A) Provision for Income Tax(B) Provision for Depreciation(C) Provision for Doubtful Debts(D) Contribution to General Reserve

13. On liquidation of a Company first payment is made in respect of—(A) Liquidator’s remuneration(B) Legal expenses(C) Preferential creditors(D) None of the above

14. Workmen’s Compensation Fund is a—(A) Provision(B) Surplus(C) Current liability(D) Loan

15. Debentures of Rs. 4,25,000 are issued against the purchase of assets of Rs. 4,50,000. In this case the amount of Rs. 25,000 is—(A) Capital Reserve(B) Securities Premium(C) Revenue Profit(D) Goodwill

16. A Company can reissue its forfeited shares—(A) At a premium(B) At a face value(C) At a discount(D) All of the above

Page 38: Commerce&Audit

17. Which of the following is an example of capital expenditure ?(A) Insurance Premium(B) Taxes and Legal expenses(C) Depreciation(D) Custom duty on import of machinery

18. Given :Average profit of a firm Rs. 21,000Normal Profit Rs. 18,000Value of goodwill on the basis of 3 years purchases of super profit shall be—(A) Rs. 39,000 (B) Rs. 9,000(C) Rs. 3,000 (D) Rs. 12,000

19. A co-operative auditor starts his work of audit from—(A) Account Books(B) Payment Books(C) Cash Book(D) None of the above

20. Accounting for research and development relates to—(A) AS-7 (B) AS-8(C) AS-9 (D) AS-10

21. Goods worth Rs. 2,000 were distributed to employees free of charge. The account to be debited is—(A) Profit and Loss A/c(B) Advertisement A/c(C) Labour Welfare A/c(D) Goods A/c

22. The maximum rate of underwriting commission on debentures is—(A) 10% (B) 2•5%(C) 12•5% (D) 5%

23. X Ltd. forfeited 20 shares of Rs. 10 each on which Rs. 6 per share were paid. If out of these shares, 8 shares were reissued to Ram as fully paid up on payment of Rs. 5•50 per share, the amount that will remain standing to the credit of Share Forfeited A/c will be—(A) Rs. 48 (B) Rs. 72(C) Rs. 84 (D) Rs. 120

24. Bonus Shares are issued to—(A) Equity Shareholders(B) Preference Shareholders(C) Debenture Holders(D) Secured Creditors

25. On an equity share of Rs. 100, the minimum amount payable on application under the law should be—

Page 39: Commerce&Audit

(A) Rs. 20 (B) Rs. 15(C) Rs. 10 (D) Rs. 5

26. Share Premium received by a Company may be used for—(A) Payment of dividend(B) Payment of remuneration to management(C) Issue of Bonus shares(D) None of these

27. Garner Vs. Murray rule applies in case of—(A) Admission of a partner(B) Dissolution of a firm(C) Retirement of a partner(D) Death of a partner

28. When sale is Rs. 4,80,000, gross loss is 25% on cost, purchase is Rs. 3,50,000 and closing stock is Rs. 60,000, the stock in the beginning would be—(A) Rs. 70,000(B) Rs. 94,000(C) Rs. 1,34,000(D) Rs. 3,50,000

29. Balance of Debenture Redemption Fund Account is transferred to—(A) Capital Reserve A/c(B) Profit and Loss A/c(C) General Reserve A/c(D) None of these

30. Planning includes—(A) Objectives(B) Policy(C) Strategy(D) All of the above

31. Hawthorne Experiments were conducted by—(A) Elton Mayo(B) Henry Fayol(C) F. W. Taylor(D) Peter F. Drucker

32. Who is called ‘The Father of Scientific Management’ ?(A) Elton Mayo(B) Henry Fayol(C) E. F. L. Breach(D) F. W. Taylor

33. A plan when expressed in quantitative terms is called—(A) Policy (B) Procedure(C) Objective (D) Budget

34. Theory ‘X’ and Theory ‘Y’ of Motivation was propounded by—(A) Herzberg

Page 40: Commerce&Audit

(B) Maslow(C) Elton Mayo(D) McGregor

35. Direction involves—(B) Physical factors(B) Financial factors(C) Human factors(D) None of the above

36. Which is the oldest form of organisation ?(A) Line(B) Line and staff(C) Functional(D) Matrix

37. Decentralisation—(A) Reduces burden of chief executives(B) Increases burden of chief executives(C) Does not effect burden (D) None of the above

38. Planning is a—(A) Primary function(B) Secondary function(C) Supplementary function(D) None of the above

39. Planning function of management is performed by—(A) Top management(B) Middle management(C) Lower management(D) All of the above

40. MBO approach in management was introduced by—(A) M. P. Follet(B) Keith Davis(C) Peter Drucker(D) Oliver Sheldon

41. Which of the following is not an element of marketing mix ?(A) Product(B) Price(C) Promotion(D) Product life cycle

42. Market segmentation means—(A) Grouping of buyers(B) Grouping of sellers(C) Grouping of middle men(D) Grouping of producers

43. Standard costing is a technique of—(A) Planning(B) Organising(C) Coordination

Page 41: Commerce&Audit

(D) Control

44. The principle of ‘Unity of Command’ was introduced by—(A) Elton Mayo(B) Henry Fayol(C) F. W. Taylor(D) Urwick

45. The last step of communication process is—(A) Controlling(B) Implementation(C) Correction(D) Reaction

46. Strategy is used in—(A) Military(B) Business(C) Industry(D) All of the above

47. ‘3-D’ Model of leadership is propounded by—(A) Likert Reinsis(B) Kurt Levis(C) William Ridin(D) Henry Fayol

48. Personal selling includes—(A) Selling(B) Services to the customers(C) Developing goodwill of the firm(D) All of the above

49. The process of evaluating employees is called—(A) Performance appraisal(B) Recruitment(C) Induction(D) Job evaluation

50. Franking machine is used for—(A) Specimen signature(B) Postage stamps(C) Arrival and departure timings(D) None of the above

51. Grapevine communication is a type of—(A) Formal communication(B) Informal communication(C) Horizontal communication(D) Vertical communication

52. The ‘Principle of Exception’ states that management should—(A) Check everything without exception(B) Invariably take corrective action without exception(C) Accept exceptions in emergencies(D) Concentrate their attention on exceptions

Page 42: Commerce&Audit

53. Which of the following is not a component of ‘SWOT Analysis’ ?(A) Threats(B) Strengths(C) Weaknesses(D) Options

54. Organisational change is necessitated by—(A) External environmental pressure(B) Internal strengths and weaknesses(C) Sequential changes(D) All the above

55. Which management functions are closely related to ?(A) Planning and Organising(B) Organising and Staffing(C) Staffing and Control(D) Planning and Control

56. Match List-I with List-II and select the correct answer from the codes given below—List-I (a) Planning (b) Staffing (c) Directing (d) Controlling List-II1. Training2. Forecasting3. Evaluating4. MotivatingCodes :(a) (b) (c) (d)(A) 2 4 1 3(B) 1 2 3 4(C) 2 1 4 3(D) 3 4 2 1

57. Memorandum of Association contains—(A) Objective clause(B) Name clause(C) Capital clause(D) All of the above

58. ‘Table A’ is an alternative to—(A) Prospectus(B) Articles of Association(C) Memorandum of Association(D) None of the above

59. The ‘Doctrine of Indoor Management’ provides protection to the—(A) Board of Directors(B) Shareholders(C) Managing Director(D) Outsiders

60. By which of the following methods a Company Secretary can be removed from his post ?(A) By passing a resolution in Board Meeting

Page 43: Commerce&Audit

(B) By order of Registrar of Companies(C) By passing a resolution in Annual General Meeting(D) None of the above

61. Dividend paid between two Annual General Meetings is known as—(A) Final dividend(B) Interim dividend(C) Special dividend(D) None of the above

62. In a Private Company there are restrictions regarding—(A) Transfer of shares(B) Issue of prospectus(C) Number of members(D) All of the above

63. The gap between two General Meetings of the Company should not be more than—(A) 12 months(B) 18 months(C) 15 months(D) 20 months

64. Statement containing details of items to be considered in a meeting is called—(A) Agenda(B) Minutes(C) Resolution(D) Notice

65. The minimum and maximum number of members in a Private Company is—(A) 2 and 50(B) 2 and 20(C) 2 and 10(D) None of the above

66. What is the maximum limit of remuneration for a whole time manager in a company ?(A) 3% of net annual profit(B) 5% of net annual profit(C) 7% of net annual profit(D) None of the above

67. Which among the following documents defines the relationship between a company and outsiders ?(A) Memorandum of Association(B) Articles of Association(C) Prospectus(D) None of the above

68. Registration is not essential in case of a—(A) Company(B) Co-operative organisation(C) Co-operative Societies(D) Business of Joint Hindu Family

69. The Meeting of Board of Directors must be called at least once—(A) In four months(B) In three months(C) In two months

Page 44: Commerce&Audit

(D) In one month

70. The Secretary of a Company should file copies of Special Resolution passed at Extraordinary General Meeting to the Registrar of Companies within—(A) 7 days (B) 14 days(C) 21 days (D) 30 days

71. What is the quorum of General Meetings in the case of a Private Company ?(A) 2 members(B) 3 members(C) 4 members(D) 5 members

72. Which of these must hold a Statutory Meeting ?(A) Public Limited Company(B) Private Company(C) Unlimited Company(D) All of these

73. When three sugar mills combine, it is an example of—(A) Vertical combination(B) Horizontal combination(C) Diagonal combination(D) None of the above

74. The first electronic computer system was developed in—(A) 1920 (B) 1940(C) 1930 (D) 1950

75. An Index is—(A) A filing system(B) A photostat procedure(C) The procedure to collect data(D) An important help in filing

76. Indian Partnership Act was passed in the year—(A) 1956 (B) 1936(C) 1932 (D) 1930

77. According to the Partnership Act, the maximum number of partners in a banking firm may be—(A) 10 (B) 12(C) 4 (D) 20

78. Dissolution of Partnership takes place when—(A) A partner misbehaves(B) A partner becomes of unsound mind(C) Business is running at a loss (D) A partner dies

Page 45: Commerce&Audit

79. The existence of a Partnership Firm is—(A) Separate from partners(B) Not separate from partners(C) Both (A) and (B)(D) None of the above

80. In which State the Mitakshara form of H.U.F. business does not apply ?(A) Rajasthan(B) Gujarat(C) West Bengal(D) Kerala

81. Letter of Credit is presented by—(A) Exporter(B) Importer(C) Custom Officer(D) Shipping Company

82. The Agreement of Partnership—(A) Must be oral(B) Must be in writing(C) Can be either oral or in writing(D) Must be in writing on a stamp paper

83. Export procedure begins with—(A) Indent(B) Shipping order(C) Marine insurance(D) None of these

84. ‘Self-help by mutual help’ is a feature of—(A) Public enterprise(B) Public corporation(C) Co-operative society(D) All of the above

85. In India private sector entered into the Life Insurance business in the year—(A) 1995 (B) 1998(C) 2000 (D) 2001

86. General Insurance Corporation of India was set up in—(A) 1956 (B) 1972(C) 1980 (D) 2000

87. ‘Salvage Charges’ is related to—(A) Life Insurance(B) Marine Insurance(C) Fire Insurance(D) None of the above

88. In ‘Life Insurance Contract’ the insurable interest is examined at the time of—

Page 46: Commerce&Audit

(A) Entering the contract(B) Filing the claim(C) Both (A) and (B)(D) Never

89. In connection with marine insurance the ‘Doctrine of Utmost Good Faith’ is based on the concept of—(A) Disclosure(B) Concealment(C) Misrepresentation(D) None of the above

90. Which statement is not true in respect of a Balance Sheet ?(A) It is an account(B) It is prepared every month(C) It is prepared to check the mathematical accuracy(D) All the above

91. A machinery is purchased for Rs. 3,00,000 and Rs. 50,000 is spent on its installation. Rs. 5,000 is spent on fuel. What will be the amount of capital expenditure ?(A) Rs. 3,00,000(B) Rs. 3,50,000(C) Rs. 3,55,000(D) None of these

92. Which of the following errors are not disclosed by Trial Balance ?(A) Compensatory Errors(B) Errors of Principle(C) Errors of Omission(D) All the three

93. A large amount spent on special advertisement is—(A) Capital Expenditure(B) Revenue Expenditure(C) Revenue Loss(D) Deferred Revenue Expenditure

94. Double Entry System was introduced in—(A) America (B) Japan(C) India (D) Italy

95. According to going concern concept a business entity is assumed to have—(A) A long life(B) A small life(C) A very short life(D) A definite life

96. The policy of ‘anticipate no profit and provide for all possible losses’ is followed due to—(A) Convention of consistency(B) Convention of conservation(C) Convention of disclosure(D) None of the above

97. Receipts and Payments Account is prepared by—(A) Companies

Page 47: Commerce&Audit

(B) Banks(C) Partnership firms(D) Non-trading organizations

98. Examination of documents and vouchers is called—(A) Physical verification(B) Test checking(C) Vouching(D) None of the above

99. Internal Audit is done by—(A) External Auditors(B) Employees of the organisation(C) Both (A) and (B)(D) Neither (A) nor (B)

100. Audit Programme is prepared by—(A) The Auditor(B) The Company(C) Internal Auditor(D) Financial Controller

101. First Auditor of a Company is appointed by the—(A) Shareholders(B) Central Government(C) Company Law Board(D) Board of Directors

102. Which of the following is not qualified to be a Company Auditor ?(A) A body corporate(B) An employee of the company(C) A person who is indebted to the company for an amount exceeding Rs. 1,000(D) All of the above

103. Which of the following statement is correct ?(A) Valuation is a part of verification(B) Verification is a part of valuation(C) Valuation has nothing to do with verification(D) None of the above

104. Test checking refers to—(A) Testing of accounting records(B) Testing of honesty of employees(C) Intensive checking of a select number of transactions(D) Checking of all transactions recorded

105. Continuous audit is suitable for—(A) Big institutions(B) Small institutions(C) General institutions(D) None of the above

106. The function of an Auditor is—(A) To examine arithmetical accuracy of accounts(B) To detect and prevent errors(C) To detect and prevent frauds

Page 48: Commerce&Audit

(D) All of the above

107. Audit is compulsory for—(A) Sole Trader(B) Partnership Firms(C) Joint Stock Companies(D) All of the above

108. Match List-I with List-II of the following and select the correct answer from answer codes given below—List-I(a) At cost price(b) At market price(c) Intrinsic value method(d) Average profit methodList-II1. Valuation of shares2. Perishable goods3. Raw material4. Valuation of GoodwillCodes :(a) (b) (c) (d)(A) 3 2 1 4(B) 1 2 3 4(C) 4 3 2 1(D) 1 4 3 2

109. In the absence of Articles of Association, an Auditor should keep in mind—(A) Prospectus(B) Table ‘A’(C) Memorandum of Association(D) Legal Declaration

110. Who may recommend Special Audit of a Company ?(A) Directors of the Company(B) Shareholders of the Company(C) Debentureholders of the Company(D) Central Government

111. Internal check is a part of—(A) Internal Audit(B) Internal Control(C) Annual Audit(D) Standard Audit

112. Cost Audit Report is to be submitted to—(A) The Company(B) The Central Government with a copy to the Company(C) The Central Government(D) The Company Secretary

113. A company auditor addresses his report to—(A) Board of Directors(B) Members(C) Managing Director(D) Company Secretary

Page 49: Commerce&Audit

114. Which of the following Sections of the Companies’ Act 1956 relates to the maintenance of proper books of accounts ?(A) Section-211(B) Section-217(C) Section-209(D) Section-205

115. X and Y are partners sharing profits in the ratio of 4 : 3. They admit a new partner Z and new profit sharing ratio is 7 : 4 : 3. The sacrificing ratio between X and Y will be—(A) Equal (B) 4 : 3(C) 2 : 1 (D) 1 : 2

116. Given :Realised value of assets Rs. 60,000Profit on Realisation Rs. 3,000Book value of assets will be—(A) Rs. 63,000(B) Rs. 57,000(C) Rs. 60,000(D) Rs. 61,500

117. A, B and C are partners sharing profits and losses in the ratio of 4 : 3 : 2. D is admitted for 1/10 share. The new ratio will be—(A) 5 : 4 : 3 : 2(B) 4 : 4 : 3 : 2(C) 4 : 3 : 2 : 1(D) None of the above

118. If actual average profit is Rs. 30,000 and normal rate of return is 12%, then capitalization value of the profits will be—(A) Rs. 3,60,000(B) Rs. 2,50,000(C) Rs. 3,05,000(D) None of the above

119. Under which Section of Companies’ Act an auditor has a right to participate and to speak in the General Meeting ?(A) 231 (B) 229(C) 226 (D) 224

120. New profit sharing ratio is calculated at the time of—(A) Admission of a new partner(B) Retirement of a partner(C) Death of a partner(D) All of the above

Answers1. (A) 2. (B) 3. (C) 4. (A) 5. (B) 6. (D) 7. (C) 8. (D) 9. (C) 10. (B)11. (B) 12. (D) 13. (C) 14. (B) 15. (A) 16. (D) 17. (D) 18. (B) 19. (C) 20. (B)21. (A) 22. (B) 23. (D) 24. (A) 25. (D) 26. (C) 27. (B) 28. (D) 29. (C) 30. (D)31. (A) 32. (D) 33. (D) 34. (D) 35. (C) 36. (A) 37. (A) 38. (A) 39. (D) 40. (C)

Page 50: Commerce&Audit

41. (D) 42. (A) 43. (D) 44. (B) 45. (D) 46. (D) 47. (C) 48. (D) 49. (A) 50. (B)51. (B) 52. (D) 53. (D) 54. (D) 55. (D) 56. (C) 57. (D) 58. (B) 59. (D) 60. (A)61. (B) 62. (D) 63. (C) 64. (A) 65. (A) 66. (B) 67. (A) 68. (D) 69. (B) 70. (D)71. (A) 72. (A) 73. (B) 74. (B) 75. (D) 76. (C) 77. (A) 78. (D) 79. (B) 80. (C)81. (B) 82. (C) 83. (A) 84. (C) 85. (C) 86. (B) 87. (B) 88. (A) 89. (A) 90. (D)91. (B) 92. (D) 93. (D) 94. (D) 95. (A) 96. (B) 97. (D) 98. (C) 99. (B) 100. (A)101. (D) 102. (D)103. (A) 104. (C) 105. (A) 106. (D) 107. (C) 108. (A) 109. (B) 110. (D)111. (B) 112. (B) 113. (B) 114. (C) 115. (D) 116. (C) 117. (C) 118. (B) 119. (A) 120. (D)

1. Consider the following statements—The vouching of remuneration to directors involve the checking of1. special resolution.2. statement of accounts.3. approval note of the Central Government.4. Minutes Book.Which of the statements given above are correct ?(A) 1 and 2 only(B) 2 and 3 only(C) 1, 2 and 4(D) 1, 3 and 4

92. Match List-I with List-II and select the correct answer using the code given below the Lists—List-I (Liability of Auditor)(a) Liability for negligence(b) Liability for misfeasance(c) Criminal liability(d) Liability to third partyList-II (Example)1. Breach of trust or duty2. Untrue statement in the prospectus3. Not liable for untrue statement in prospectus4. Dividends paid out of capitalCodes :(a) (b) (c) (d)(A) 4 3 2 1(B) 2 1 4 3(C) 4 1 2 3(D) 2 3 4 1

93. Consider the following statements—1. Corporate governance requires the appointment of audit committees by publiclimited companies.2. The members of an audit committee should be those non-executive directors who do not represent the controlling group.3. The independence of the statutory auditors is likely to be affected if he deals directly with the audit committee.Which of the statements given above are correct ?(A) 1 and 2 only(B) 2 and 3 only(C) 1 and 3 only(D) 1, 2 and 3

94. Under the head 'Secured Loans' the following are disclosed—1. Loans and Advances from banks.2. Debentures.3. Loans and Advances from subsidiaries.4. Other loans and advances.What is the correct chronological order of disclosure of the items given above in balance sheet of a company ?(A) 3 – 1 – 2 – 4(B) 2 – 4 – 3 – 1(C) 3 – 4 – 2 – 1(D) 2 – 1 – 3 – 4

95. Consider the following statements—Accounting control comprises1. The plan of organization concerned with decision process.2. The plan of organization concerned with safeguarding of assets.3. The procedure concerned with safeguarding of assets.Which of the statements given above are correct ?(A) 1 and 2 only(B) 2 and 3 only(C) 1 and 3 only

Page 51: Commerce&Audit

(D) 1, 2 and 3

96. Which of the following statements is/are correct ?1. Internal control includes quality control.2. Internal control is compulsory for all forms of business organizations.3. Internal control helps external auditor to design suitable audit programme for client's organization.Select the correct answer using the code given below—(A) 1 only(B) 2 only(C) 3 only(D) 1 and 3

97. Which of the following is not a part of the usual contents of the current file and audit working papers ?(A) A copy of accounts on which auditors are reporting(B) Audit programme(C) Minutes of the meetings(D) Memorandum of Association and Articles of Association

98. Consider the following statements—Audit working papers are maintained because1. they act as a guide to subsequent audit.2. they verify the arithmetical accuracy of the books of accounts.3. they provide evidence of the work performed by the auditor.Which of the statements given above are correct ?(A) 1 and 2 only(B) 2 and 3 only(C) 1 and 3 only(D) 1, 2 and 3

99. Match List-I with List-II and select the correct answer using the code given below the Lists—List-I (Technique)(a) Rotation of Duties(b) Random Sampling(c) Trend Analysis(d) Cash verificationList-II (Auditing Procedure)(1) Internal control(2) Physical examination(3) Selective verification(4) Overall assessmentCodes :(a) (b) (c) (d)(A) 1 4 3 2(B) 2 3 4 1(C) 1 3 4 2(D) 2 4 3 1

100. Consider the following stages an auditor has to undergo before commencing an audit—1. Ascertaining the scope of audit work.2. Gaining knowledge about business of the client.3. Going through the agreement with the client.4. Instructions to and information from the client.What is the correct sequence of the above stages ?(A) 2 – 1 – 3 – 4(B) 3 – 4 – 2 – 1(C) 2 – 4 – 3 – 1(D) 3 – 1 – 2 – 4

101. Which one of the following statements is correct ?(A) If an asset is used by a company, it should be owned by the company(B) Verification of liabilities means that the amounts of current and long-term liabilities shown in the balance sheet are correct(C) Vouching and verification means the same thing(D) It is not the duty of an auditor to confirm the physical existence and ownership of an asset

102. The following steps should be adopted by an auditor for verification of land—1. To examine that the land has been shown in the balance sheet at its original cost.2. To ensure that the cost of land also includes the expenses like broker's commission, legal expenses, improvement expenses etc.3. To get a certificate from solicitor of the client regarding validity of the title deeds.4. To examine that the land is not subject to any charge or mortgage.What is the correct sequence of the above steps to be followed by an auditor during the course of auditing ?(A) 1 – 4 – 2 – 3(B) 3 – 2 – 4 – 1

Page 52: Commerce&Audit

(C) 1 – 2 – 4 – 3(D) 3 – 4 – 2 – 1

103. What does payout ratio mean ?(A) Ratio of debtors to creditors(B) Ratio of profit distributed to profit retained(C) Ratio of dividends per share and earnings per share(D) Retained earnings

104. Consider the following statements—1. Premium on issue of shares is transferred to General Reserve Account.2. For declaration of bonus shares out of General Reserve, a resolution in the shareholders' meeting is necessary.Which of the statements given above is/are correct ?(A) 1 only(B) 2 only(C) Both 1 and 2(D) Neither 1 nor 2

105. How is 'Calls in arrear' shown in the balance sheet on the liability side ?(A) By deducting the amount from the called up capital(B) Under the heading 'Reserves and Surplus'(C) By deducting the amount from 'Capital Reserve'(D) By deducting the amount from the 'Share Forfeited Account'

106. Match List-I with List-II and select the correct answer using the code given below the Lists—List-I (Activity)(a) Admission of a partner(b) Partnership dissolution(c) Rectification of errors(d) Transfer of realization profitList-II (Account)(1) Capital Accounts(2) Suspense Account(3) Revaluation Account(4) Realization AccountCodes :(a) (b) (c) (d)(A) 3 4 2 1(B) 2 1 3 4(C) 3 1 2 4(D) 2 4 3 1

107. The balance of old provision for doubtful debts on 1-4-2006 was Rs. 10,000. The bad debts written off during the year 2006-07 amounted to Rs. 12,000, and the new provision requiredon 31-3-2007 was Rs. 15,000. What is the total amount to be debited to profit and loss account on account of bad debts and the provision for doubtful debts ?(A) Rs. 37,000(B) Rs. 27,000(C) Rs. 17,000(D) Rs. 15,000

108. A and B sharing profit in the ratio of 3 : 2 and having capitals of Rs. 30,000 (for A) and Rs. 15,000 (for B), decided to dissolve their firm. After paying off all liabilities, cash realized from various assets is Rs. 15,000. How will this amount be distributed to A and B ?(A) A–Rs. 9,000 and B–Rs. 6,000(B) A–Rs. 10,000 and B–Rs. 5,000(C) A–Rs. 7,500 and B–Rs. 7,500(D) A–Rs. 12,000 and B–Rs. 3,000

109. X and Y sharing profits in the ratio of 7 : 3 admit Z on 37th share in the new firm. Z takes 27 th from X and 17th from Y.What is the new ratio among X, Y and Z ?(A) 7 : 3 : 3(B) 4 : 2 : 15(C) 14 : 6 : 15(D) 29 : 11 : 30

110. Given—Opening inventory : Rs. 3,500Closing inventory : Rs. 1,500Cost of goods sold : Rs. 22,000What is the amount of purchase ?(A) Rs. 20,000(B) Rs. 24,000

Page 53: Commerce&Audit

(C) Rs. 27,000(D) Rs. 17,000

111. Consider the following—Amortization refers to writing off the value of(1) tangible assets(2) intangible assets(3) fictitious assetsWhich of the above is/are correct ?(A) 1 and 2(B) 2 and 3(C) 1 only(D) 2 only

112. The following information is disclosed by 'A'— Rs. Provision for doubtful debts as on 1.1.2006 : 6,000 Bad debts written off during the year 2006 : 1,200 Total debtors as on 31.12.2006 : 80,000 A provision for Doubtful Debts to be made @ 5% What is the amount to be shown on the credit side of P/L A/c ?(A) Rs. 4,800(B) Rs. 2,400(C) Rs. 1,200(D) Rs. 800

113. What is reduction of paid-up share capital called ?(A) External reconstruction(B) Internal reconstruction(C) Reorganization(D) Redemption of capital

114. Under which of the following methods of depreciation the amount of an asset is never reduced to zero ?(A) Straight line method(B) Diminishing balance method(C) Sums of Years Digit method(D) Annuity method

115. An insurance claim of Rs. 300 was accepted in respect of stock (inventory) of Rs. 500, which was destroyed by fire. Rs. 200 not covered by insurance should be debited to which one of the following ?(A) Stock account(B) Trading account(C) Profit and loss account(D) Goodwill account

116. Debtors turnover ratio is 4.What is the average collection period ?(A) 5 months(B) 4 months(C) 3 months(D) 2 months

117. The net profits of a business after providing for taxation for the past five years are Rs. 80,000, Rs. 85,000, Rs. 92,000, Rs. 1,05,000 and Rs. 1,18,000. Capital employed in the business is Rs. 8,00,000. Normal rate of return is 10%. What is the value of goodwill on the basis of capitalization of super profit method ?(A) Rs. 1,00,000(B) Rs. 1,50,000(C) Rs. 1,60,000(D) Rs. 1,80,000

118. Given—Purchases — Rs.50,000Sales — Rs.90,000Closing Stock — Rs.7,000Manufacturing Exp. — Rs.5,000Rate of gross profit — Rs.3313% on costWhat is the opening stock ?(A) Rs. 20,000(B) Rs. 19,500(C) Rs. 22,500(D) Rs. 25,000

119. A particular firm provided the following data for a year—Current Ratio 2•5 : 1Liquid Ratio 1•5 : 1Net Working CapitalRs. 3,00,000

Page 54: Commerce&Audit

What are the current assets and current liabilities of this firm, respectively ?(A) Rs. 3,00,000 and Rs. 1,50,000(B) Rs. 5,00,000 and Rs. 2,00,000(C) Rs. 2,00,000 and Rs. 5,00,000(D) Rs. 3,00,000 and Rs. 1,00,000

120. Depreciation is a process of which one of the following ?(A) Valuation of assets(B) Allocation of acquisition cost over the estimated useful life of the asset(C) Allocation of realizable value over the estimated useful life of the asset(D) Estimating the market value of the asset on the balance sheet date

Answers

1. (B) 2. (C) 3. (A) 4. (B)

5. (A) Small shareholders of a public company will be able to appoint (elect) one director on the Board of Company if the company has a paid up capital of Rs. 5 crore or more, or has 1000 or more small shareholders as per section 252 Companies (Amendment) Act, 2000.

6. (C) 7. (A)

8. (D) 7S McKinsey Model's elements includes :(i) Strategy, (ii) Structure, (iii) System, (iv) Shared value, (v) Skills, (vi) Style, (vii) Staff

9. (C) 10. (D) 11. (C)

12. (C) Funds from operation computed as follows :Net profit — Rs.2,00,000Add : Preliminary Exp. — Rs.20,000Depreciation — Rs.40,000= 2,60,000Less : Gain on sale of machinery — Rs.10,000Funds from operation = 2,50,000

13. (C) 14. (C) 15. (D) 16. (A) 17. (A) 18. (C) 19. (C)

20. (A) According to Spicer and Pegler internal check is an arrangement of staff duties whereby no one person is allowed to carry through and record every aspect of a transaction so that, without collusion between any two or more persons, fraud is prevented and at the same time the possibilities of errors are reduced to a minimum.

21. (B) 22. (C) 23. (C) 24. (C) 25. (B) 26. (D) 27. (D) 28. (D) 29. (B) 30. (D)31. (A) 32. (D) 33. (B) 34. (B) 35. (B) 36. (B) 37. (A) 38. (C) 39. (A) 40. (D)41. (B) 42. (A) 43. (C) 44. (B) 45. (C) 46. (C) 47. (C) 48. (C) 49. (A) 50. (B)51. (D) 52. (B) 53. (D) 54. (C) 55. (C) 56. (A) 57. (D) 58. (D) 59. (A) 60. (B)61. (B) 62. (C) 63. (A) 64. (B) 65. (C) 66. (C) 67. (D)

68. (C) An organisation in which each employee reports to both a functional manager and to a project manager as Matrix Structure of organisation.

69. (D) 70. (B) 71. (B) 72. (B) 73. (C) 74. (B) 75. (D) 76. (A) 77. (A) 78. (C)79. (B) 80. (B) 81. (C) 82. (A) 83. (C) 84. (B) 85. (B) 86. (A) 87. (C) 88. (D)89. (B) 90. (A) 91. (D) 92. (C) 93. (A) 94. (D) 95. (B) 96. (D) 97. (D) 98. (C)99. (C) 100. (D) 101. (B) 102. (B) 103. (C) 104. (B) 105. (A) 106. (A) 107. (C)108. (A) 109. (D) 110. (A) 111. (D) 112. (D) 113. (B) 114. (B) 115. (C) 116. (C)117. (C) 118. (B) 119. (B) 120. (B)

Page 55: Commerce&Audit

R.A.S./R.T.S. (Pre.) Examination, 2008

Public Administration : Solved Paper(Held on 7-1-2009)

1. Which of the following theories focuses on informal organization and participative management ?(A) Human Relations Theory(B) Scientific Management Theory(C) Behavioural Theory(D) Contingency TheoryAns : (A)

2. “Until administrative description reaches a higher level of sophistication, there is little reason to hope that rapid progress will be made towards the identification and verification of valid administrative principles.” Who said it ?(A) Henri Fayol(B) Leonard White(C) Herbert Simon(D) Robert DahlAns : (D)

3. Which of the following attempts to construct a theory of public organization in tune with consumer interests and individual preferences ?(A) Development Administration Approach(B) Public Choice Approach(C) Co-optation Concept(D) Behavioural ApproachAns : (B)

4. Who theorized that ‘Every person has certain basic assumptions about other people’s attitude toward work and organization.’ ?(A) Elton Mayo(B) Frederick Herzberg(C) Chris Argyris(D) Douglas McGregorAns : (D)

5. Which of the following has been termed as the biological parent of Public Administration ?(A) Sociology(B) Political Science(C) Business Administration(D) PsychologyAns : (B)

6. The central theme of Comparative Public Administration is—(A) Administrative structure(B) Bureaucracy(C) Citizen-Administration Relations(D) LeadershipAns : (B)

7. Match the List-I with List-II and select the correct answer :List-I (Theme)

Page 56: Commerce&Audit

(a) Job Enrichment(b) Mechanical Study of the Process(c) Human Dimensions(d) Social Equity AttitudeList-II (Thinker)1. D. Waldo2. W. F. Willoughby3. F. Herzberg4. Herbert SimonCodes :(a) (b) (c) (d)(A) 3 2 1 4(B) 4 3 2 1(C) 3 2 4 1(D) 2 4 1 3Ans : (A)

8. The most preferred perspective to distinguish public and private in administration is—(A) Agency(B) Interest(C) Access(D) CommitmentAns : (B)

9. “Lack of courage to delegate properly and of knowledge how to do it is one of the most general causes of failure in organization.” Who said it ?(A) Henri Fayol(B) Lyndall Urwick(C) Paul H. Appleby(D) Dwight WaldoAns : (A)

10. Who was the Chairman of the Comparative Administration Group 1963 ?(A) Herbert Simon(B) Robert Dahl(C) Fred Riggs(D) Franc MariniAns : (C)

11. Politicisation of bureaucracy means—(A) Responding to the will of the government of every complexion(B) Sacrificing professional norms for party consideration(C) Supporting party ideology(D) Attending party assembliesAns : (B)

12. “In the past the man was first. In the future the system will be the first.” Who said it ?(A) Frederick Taylor(B) Peter Drucker(C) Marshall Dimock(D) Leonard WhiteAns : (B)

13. Rensis Likert’s System I is characterised by—(A) Supportive Leadership(B) Low Motivation

Page 57: Commerce&Audit

(C) Self Regulation(D) OpennessAns : (B)

14. The basic premise of New Public Management is—(A) Governments as the primary agent of all social action(B) Large governmental bureaucracies(C) More flexible and open governments(D) Paternal governmentsAns : (C)

15. Match the List-I with List-II and select the correct answer :List-I(a) Direct Authority(b) Delegated Authority(c) Functional Authority(d) Personal AuthorityList-II1. Seniority2. Unbroken line between Issuer and Acceptor3. An Intermediate Agency between Issuer and Acceptor4. Intrinsic by special knowledgeCodes :(a) (b) (c) (d)(A) 1 2 3 4(B) 2 1 4 3(C) 2 3 4 1(D) 1 3 2 4Ans : (C)

16. According to Mary Parker Follett, the principles of effective coordination are—(a) Intellectual Ability(b) Continuity(c) Reciprocity(d) Mutual ConsultationCodes :(A) (a) and (b)(B) (a), (b) and (c)(C) (b) and (c)(D) (b), (c) and (d)Ans : (C)

17. Dwight Waldo in his book ‘The Administrative State’ has attacked—(a) Notion of Unchanging Principles of Administration(b) Human Emotions in Organization(c) Narrowness of Values of Economy and Efficiency(d) Inconsistencies in Methodology used in determining the principlesCodes :(A) (a), (b) and (d)(B) (b), (c) and (d)(C) (a), (c) and (d)(D) (a), (b) and (c)Ans : (C)

18. Human Relations Theory believed that the people are—(A) Economic Men

Page 58: Commerce&Audit

(B) Homogeneous(C) Heterogeneous(D) AtomisticAns : (C)

19. Match the List-I with List-II and select the correct answer :List-I(a) Substantive Coordination(b) Internal Coordination(c) Horizontal Coordination(d) Procedural CoordinationList-II1. Between Units in the same level2. Between different units within the organisation3. Generalised description of the behaviour and relationship of the members of the organization4. Content of the organization’s activitiesCodes :(a) (b) (c) (d)(A) 4 2 1 3(B) 1 4 2 3(C) 3 4 1 2(D) 2 1 3 4Ans : (A)

20. The linking process for interaction under systems approach does not include—(A) Communication(B) Delegation(C) Balance(D) Decision MakingAns : (B)

21. “Work division is the foundation of organization, indeed, the reason for organization.” Who said it ?(A) Frederick Taylor(B) Henri Fayol(C) Luther Gulick(D) Mary Parker FollettAns : (C)

22. The semantic barrier to communication is not—(A) Environment(B) Distracting Noise(C) Distance(D) Poor Listening HabitsAns : (C)

23. The Bills which can be introduced but cannot be considered without the prior recommendation of the President is—(A) Money Bills under Article 110(B) Financial Bills not consisting solely of matters specified in Article 110(C) Ordinary Bills involving expenditure from the Consolidated Fund(D) Bills affecting taxes in which States are interestedAns : (C)

24. New Public Administration shares this feature with Development Administration—(A) Effective coordination(B) Change orientation

Page 59: Commerce&Audit

(C) Temporal Dimension(D) Ecological PerspectiveAns : (B)

25. “As long as the study of Public Administration is not comparative, claims for a science of Public Administration is rather hollow.” Who said it ?(A) Fred Riggs(B) Max Weber(C) Robert Dahl(D) Ferrel HeadyAns : (C)

26. In Britain the ‘Priestly formula’ relates to—(A) Recruitment(B) Promotion(C) Pay(D) Disciplinary ActionAns : (B)

27. Whitley Council in UK is not a forum for—(A) Discussion and Negotiation(B) Ventilation of Grievances(C) Settlement of Disputes on Service Conditions(D) Settlement of Individual cases relating to Promotion & Disciplinary ActionAns : (D)

28. “Neutral bureaucracy can be found only in a secular political culture.” Who said it ?(A) M. E. Dimock(B) Glenn Stahl(C) G. A. Almond(D) Max WeberAns : (C)

29. Match the List-I with List-II and select the correct answer :List-I (Concept)(a) Monte Carlo(b) Gambler(c) Synetics(d) ContractList-II (Meaning)1. Generation of Alternative Solutions2. Narrow Form of Simulation3. Formal Basis of Authority4. To take Greater RiskCodes :(a) (b) (c) (d)(A) 2 4 1 3(B) 2 1 3 4(C) 1 4 2 3(D) 4 1 3 2Ans : (C)

30. The most effective means of citizens’ control over administration is—(A) Elections(B) Pressure Groups(C) Advisory Committees

Page 60: Commerce&Audit

(D) Public OpinionAns : (A)

31. The U.S. President’s principal staff agency in fiscal, legislative and administrative management is—(A) Bureau of the Budget(B) The Office of Management and Budget(C) Department of the Treasury(D) White House OfficeAns : (B)

32. Which of the following is not a feature of U.S. personnel system ?(A) Competitive Service(B) Lateral Entry(C) Post-entry Training(D) Rule of ThreeAns : (B)

33. Grants from Centre to the States are regulated by the following Articles of the Constitution—(A) 268, 269, 270, 272(B) 270, 271, 274(C) 273, 275, 282(D) 275, 282, 287Ans : (A)

34. The expectancy theory of motivation depends upon the employee’s perception of the relationship between—(A) Need, Performance and Achievement(B) Effort, Performance and Reward(C) Desire, Performance and Response(D) Preference, Performance and ActionAns : (B)

35. Who referred the interaction between politicians and administrators as ‘administrative politics’ ?(A) Michael Murray(B) Michael Crozier(C) David Rosenblom(D) Peter SelfAns : (D)

36. The techniques applied in performance appraisal in U.S.A. are—(a) Rating Scale(b) Critical Incidents(c) Forced choice(d) Forced DistributionCodes :(A) (a) and (b)(B) (c) and (d)(C) (a), (b) and (c)(D) (a), (b), (c) and (d)Ans : (D)

37. The political rights granted to the Civil Servants in France are—(a) Right to become member of a political party(b) Right to participate in political activities(c) Right to criticise government’s policies(d) Right to contest elections to any representative office

Page 61: Commerce&Audit

Codes :(A) (a) and (b)(B) (a), (b) and (d)(C) (c) and (d)(D) (a), (b), (c) and (d)Ans : (D)

38. The functions performed by the then Secretary to the Governor-General before Independence are now performed by—(A) Chief Secretary(B) Cabinet Secretariat(C) Prime Minister’s Office(D) Ministry of Home AffairsAns : (C)

39. Match the List-I with List-II and select the correct answer :List-I(a) Economic Man(b) Decision Tree(c) Link Pin Concept(d) SimulationList-II1. Trial and Error Approach to Complex Problems2. Subordinate becomes a Member of the Superior’s Decision Unit3. Optimum Decision4. Graphic Method to see Alternative SolutionsCodes :(a) (b) (c) (d)(A) 3 4 2 1(B) 2 1 3 4(C) 3 4 1 2(D) 1 3 4 2Ans : (A)

40. In USA the authority to regulate the formation and dissolution of Federal Departments is vested in—(A) President(B) Congress(C) Constitution(D) President in consultation with the Secretary of StateAns : (B)

41. Match the List-I with List-II and select the correct answer :List-I(a) Zone of Acceptance(b) Zone of Indifference(c) Span of Attention(d) Scalar chainList-II1. Graicunas2. Henri Fayol3. Herbert Simon4. Chester BarnardCodes :(a) (b) (c) (d)(A) 2 3 4 1(B) 3 4 1 2

Page 62: Commerce&Audit

(C) 4 3 2 1(D) 3 2 1 4Ans : (B)

42. Which of the following, according to Bruce Stone, is not a ‘Bottom-Up’ concept of accountability ?(A) Parliamentary control(B) Judicial and Quasi-Judicial Review(C) Constituency Relationship(D) MarketAns : (B)

43. Match the List-I with List-II and select the correct answer :List-I(a) Civil Service Commission(b) Recruitment by Promotion(c) Open Competition(d) Centralised RecruitmentList-II1. 19172. 19263. 19194. 1833Codes :(a) (b) (c) (d)(A) 2 3 1 4(B) 2 1 4 3(C) 3 1 2 4(D) 4 3 1 2Ans : (B)

44. The concept of Citizens’ Charter as a means to greater accountability and openness was first introduced in 1991 in—(A) USA(B) U.K.(C) France(D) IndiaAns : (B)

45. Which of the following is not an informal training type ?(A) Training by Communication(B) Conference Method(C) Departmental Training(D) Syndicate MethodAns : (D)

46. A grant for meeting an unexpected demand is called—(A) Supplementary Grant(B) Votes on Account(C) Votes on Credit(D) Exceptional GrantAns : (D)

47. “Budget is a document which is unstitched and reassembled by the Appropriation Committees of both the Houses.” This statement of Aaron Wildrasky applies to the budget of—(A) United States of America(B) United Kingdom

Page 63: Commerce&Audit

(C) France(D) IndiaAns : (A)

48. The best check on bureaucratic power, according to Weber, lies with—(A) Powerful Private Groups(B) Strong Bureaucratic leadership(C) Strong Parliament(D) Superior Technical Knowledge of the Chief ExecutiveAns : (D)

49. ‘If a society is not at all differentiated, if there are no specialists, if everyone can do everything.’ Riggs calls it a—(A) Society(B) Prismatic Society(C) Diffracted Society(D) Transitional SocietyAns : (B)

50. Which one is not correctly matched ?(A) Compromiser—poor decision maker(B) Executive—good motivator(C) Bureaucrat—control of situation by rules(D) Developer—interest in harmony, avoids conflictsAns : (B)

51. Match the List-I with List-II and select the correct answer :List-I (Books)(a) The New Science of Management decision(b) Organization and Innovation(c) The Achieving Society(d) Eupsychian ManagementList-II (Author)1. David McClelland2. Abraham Maslow3. Chris Argyris4. Herbert SimonCodes :(a) (b) (c) (d)(A) 1 2 3 4(B) 3 1 4 2(C) 4 3 1 2(D) 4 2 3 1Ans : (C)

52. ‘The bureaucracy is a circle, which no one can escape, its hierarchy is a hierarchy of knowledge.’ Who said it ?(A) Max Weber(B) Karl Marx(C) Blau and Scott(D) T. ParsonsAns : (B)

53. Which of the following is an area of Planning Commission ?(A) Review of the Union and State Finances(B) Maintaining Macro-economic Stability

Page 64: Commerce&Audit

(C) Public and Foreign Loans(D) Suggesting Ways and Means to Restructure Public FinanceAns : (A)

54. The advantages of a departmental undertaking are—(a) Optimum Utilisation of Financial Resources(b) Operational Flexibility(c) High Degree of Accountability(d) Autonomy in workingCodes :(A) (a) and (b)(B) (a) and (c)(C) (a), (b) and (d)(D) (b), (c) and (d)Ans : (B)

55. Match the List-I with List-II and select the correct answer :List-I(a) Endo-prismatic(b) Exo-prismatic(c) Ortho prismatic(d) DiffractedList-II1. Semi differentiated and malintegrated2. Congruence of aspiration and reality3. Impetus from outside4. Internally generated movement towards diffractionCodes :(a) (b) (c) (d)(A) 3 2 4 1(B) 4 3 1 2(C) 2 4 3 1(D) 1 2 3 4Ans : (B)

56. The authority which can make rules for the transaction of the business of the Government of India is—(A) President(B) Prime Minister(C) Parliament(D) Ministry of Home AffairsAns : (A)

57. Match the List-I with List-II and select the correct answer :List-I (Concept)(a) Knowledge Worker(b) Domination by Organization personality(c) Action-centred Leadership Theory(d) Organization as BureaucracyList-II (Thinkers)1. John Adair2. Max Weber3. Chester Barnard4. Peter DruckerCodes :(a) (b) (c) (d)(A) 1 2 3 4

Page 65: Commerce&Audit

(B) 1 3 2 4(C) 4 3 1 2(D) 4 3 2 1Ans : (C)

58. ‘No money shall be withdrawn from the Consolidated Fund except under appropriation made by law.’ This has been provided in—(A) Article 110(B) Article 112(C) Article 113(D) Article 114Ans : (D)

59. Match the List-I with List-II and select the correct answer :List-I(a) All India Services(b) Classification of Civil Services(c) Regulation of Recruitment and Conditions of Service(d) General Policies Regarding all Central ServicesList-II1. Parliamentary Act2. Ministry of Personnel, Public Grievances and Pensions3. Article 3124. Civil Service Rules 1930Codes :(a) (b) (c) (d)(A) 1 2 3 4(B) 3 4 1 2(C) 3 2 4 1(D) 1 2 4 3Ans : (B)

60. “Increasing efficiency must begin with those higher ups. This is the essence of scientific management.” Who said it ?(A) Woodrow Wilson(B) Louis Brandis(C) Frederick Taylor(D) Harrington EmersonAns : (D)

61. The conditions necessary to ensure that neutrality does not lead to a state of inertia and status-quo maintenance are—(a) cultural or class congruence between Ruling group and Administrative elite(b) absence of fundamental disagreement over social core values(c) active participation in political process(d) a shared belief system’s presenceCodes :(A) (a) and (b)(B) (a), (c) and (d)(C) (a), (b) and (c)(D) (a), (b) and (d)Ans : (D)

62. Rigg’s concept of development considers penetration as a factor of integration. Consider the following—(a) The ability of the government to make and carry out decisions.

Page 66: Commerce&Audit

(b) People’s receptivity to law(c) People’s will to participate(d) People’s ability to participateWhich of the above is correct regarding penetration ?Codes :(A) (a)(B) (b) and (d)(C) (c)(D) (c) and (d)Ans : (A)

63. Who recommended that the National Development Council should be given a constitutional status under Article 263 and renamed as National Economic and Development Council ?(A) Administrative Reforms Commission(B) Sarkaria Commission(C) P. J. Rajamannar Committee(D) Chief Secretary’s ConferenceAns : (B)

64. What falls within the purview of consultation of Union Public Service Commission ?(a) Direct Appointment(b) Promotion and Transfer(c) Pay and Service Conditions(d) Classification and TrainingCodes :(A) (a) and (b)(B) (b) and (c)(C) (a), (b) and (c)(D) (b), (c) and (d)Ans : (A)

65. In a State the Chief Secretary acts as—(a) Principal Adviser to the Governor(b) Coordinating link in inter-State Disputes(c) Secretary to the Chief Minister(d) Chairman of all Planning and Development Coordination CommitteesCodes :(A) (a) and (c)(B) (c) and (d)(C) (a), (b) and (c)(D) (b), (c) and (d)Ans : (B)

66. Which of the following provided for the appointment of the same person as Governor of two or more States ?(A) Article 153(B) Article 155(C) 7th Constitutional Amendment(D) 11th Constitutional AmendmentAns : (C)

67. Audit of State Government Accounts is a—(A) State Subject(B) Union Subject(C) Subject on Concurrent List(D) Subject of Fiscal Policy

Page 67: Commerce&Audit

Ans : (B)

68. The body that examines the expenditure of the Departments and questions the propriety of these expenditures is—(A) Public Accounts Committee(B) Estimate Committee(C) Comptroller and Auditor General(D) Committee on AssurancesAns : (C)

69. Reappropriation is permissible only—(A) Between voted and charged items of expenditure(B) Between different grants voted by the Lok Sabha(C) Within the same grant only(D) To meet any expenditure not already sanctioned by the Lok SabhaAns : (C)

70. It is a comprehensive account of the receipts and expenditure of the Government under various heads and sub-heads of the Budget—(A) The Appropriation Account(B) The Finance Account(C) Economic Classification of the Budget(D) Functional Classification of the BudgetAns : (B)

71. Which of the following statements are true about Lokayukta in Rajasthan ?(a) He is appointed by the Governor and is responsible to the legislature.(b) His jurisdiction spans over Ministers, Members of State Legislature and higher Civil Servants.(c) He considers the cases of corruption and mal administration.(d) His job is confined to investigate allegations and not grievances.Codes :(A) (a) and (d)(B) (a), (b) and (c)(C) (a), (b), (c) and (d)(D) (a) and (c)Ans : (D)

72. Among the critical dimensions of the leadership situation the most important dimension, according to Fiedler, is—(A) Position Power(B) Task Structure(C) Leader-Member Relations(D) Least Preferred Co-workerAns : (D)

73. Match the List-I with List-II and select the correct answer :List-I(a) Citizen’s Charter(b) Right to Information(c) Judicial Control(d) Legislative ControlList-II1. Total Quality Approach2. Accountability for Decision3. Transparency4. Guarantee against Arbitrariness

Page 68: Commerce&Audit

Codes :(a) (b) (c) (d)(A) 3 4 1 2(B) 1 2 3 4(C) 3 1 2 4(D) 1 3 4 2Ans : (D)

74. Which one of the following is not a function of the Directorate ?(A) Formulation of the Budget of the Department(B) Inspection of the execution of work by Field Officers(C) Rendering technical advice to the Minister/Secretary(D) Coordination of interdepartmental functionsAns : (A)

75. Which of the following is not correctly matched ?(A) Injunction—Strong arm of equity(B) Quo Warranto—Prevents illegal assumption of Public Office(C) Prohibition—Issued against administrative authorities(D) Habeas Corpus—The bulwark of individual liberty.Ans : (C)

76. The recommendation to the Union Government for the grant of financial assistance to the States is made by—(a) The President(b) The Comptroller and Auditor General of India(c) The Finance Commission(d) The Planning CommissionCodes :(A) (a) and (c)(B) (b) and (c)(C) (c) and (d)(D) (b), (c) and (d)Ans : (C)

77. “It transforms the representative democracy into participatory democracy.” This statement has a reference to—(A) Parliamentary System of the Government(B) Article 40 of the Constitution(C) 73rd Constitutional Amendment(D) 74th Constitutional AmendmentAns : (C)

78. Which of the following is not a corporation form ?(A) Unit Trust of India(B) Indian Oil Corporation(C) Food Corporation of India(D) State Bank of IndiaAns : (B)

79. Which of the following statements are correct about the National Development Council ?(a) It symbolises a federal approach to planning.(b) It was set up by a proposal of the Cabinet Secretariat in August 1952.(c) It was reconstituted in 1968 on the recommendation of the Administrative Reforms Commission.(d) The initiative to convene its meeting lies with the Planning Commission.Codes :

Page 69: Commerce&Audit

(A) (a) and (b)(B) (a), (b) and (c)(C) (a), (b) and (d)(D) (a), (b), (c) and (d)Ans : (C)

80. For which form of Public Enterprise it is said that “It is a child of the State which grows into maturity as soon as it is born” ?(A) Departmental Form(B) Company Form(C) Corporation Form(D) Commission FormAns : (C)

81. Voluntary provisions of 73rd Constitutional Amendment include—(A) Three Tier Panchayati Raj Institutions(B) Reservation for Backward Classes(C) Reservation for Women up to 1/3 seats(D) Five Year term for PRIsAns : (B)

82. Which of the following is the Central Personnel Agency in India ?(A) Union Public Service Commission(B) Ministry of Home Affairs(C) Department of Personnel and Training(D) Staff Selection BoardAns : (C)

83. An urban body in the modern sense was first established in the year—(A) 1660(B) 1687(C) 1951(D) 1992Ans : (B)

84. It is not created by an act of State or Union Legislature—(A) Municipal Corporation(B) Notified Area Committee(C) Town Area Committee(D) Cantonment BoardAns : (B)

85. The Central Council of Local Government—(a) is an advisory body.(b) is constituted under Article 263(c) Consists of Union Minister for Urban Development and State Ministers for Local Self Government(d) Controls the Central financial assistance to local bodiesCodes :(A) (a) and (c)(B) (b) and (d)(C) (a), (b) and (c)(D) (b), (c) and (d)Ans : (C)

86. The Powers, Authority and Responsibilities of Panchayats in India are mentioned in the Article—(A) 243 B

Page 70: Commerce&Audit

(B) 243 D(C) 243 G(D) 243 WAns : (C)

87. Match the List-I with List-II and select the correct answer :List-I(a) Miniature District(b) King-pin of Revenue Administration(c) Unit for Revenue and Criminal Administration(d) First Line Supervisor in the chain of Revenue AdministrationList-II1. Sub Division2. Kanungo3. Tehsil4. PatwariCodes :(a) (b) (c) (d)(A) 1 2 3 4(B) 1 4 3 2(C) 3 2 4 1(D) 3 4 1 2Ans : (D)

88. Gram Sabha has been called “Embodiment of Direct Democracy” by—(A) Balwantray Mehta Committee(B) Ashok Mehta Committee(C) G. V. K. Rao Committee(D) L. M. Singhvi CommitteeAns : (D)

89. The transitional area, smaller urban area, larger urban area are differentiated on the basis of—(a) population and its density(b) level of per capita income(c) revenue generated for local administration(d) percentage of employment in non-agricultural activitiesCodes :(A) (a) and (c)(B) (a), (b) and (c)(C) (c) and (d)(D) (a), (c) and (d)Ans : (D)

90. With the advent of development administration, the Collector has emerged as—(A) Symbol of State Power(B) Head of General Administration(C) Coordinator of Development Programmes(D) Executive Head of Development ProgrammesAns : (C)

Directions—(Q. 91–100) The following consist of two statements, one labelled as Assertion (A) and the other as Reason (R). You are to examine these two statements carefully and select the answers to these questions using the codes given below—(A) Both (A) and (R) are true and (R) is the correct explanation of (A)(B) Both (A) and (R) are true, but (R) is not the correct explanation of (A)(C) (A) is true, but (R) is false

Page 71: Commerce&Audit

(D) (A) is false, but (R) is true

91. Assertion (A) : The study of Public Administration lacks cross cultural concepts and principles for being able to be recognised as science.Reason (R) : Administrative culture cannot transcend the national frontiers and national experiences.Ans : (C)

92. Assertion (A) : The Contingency approach and the Transactional approach, both are mutually complementary and quite interrelated.Reason (R) : Unlike the Contingency models, which concentrate only on the problems confronting the leader in dealing with a small group, Transactional approaches also analyse the leader’s subordinates and the problems confronting them.Ans : (B)

93. Assertion (A) : Like any social science, Public Administration has its own corpus of methodologies.Reason (R) : Public Administrationists are aware that the central methodologies of the ‘mother discipline’ usually are irrelevant of practising Public Administration.Ans : (A)

94. Assertion (A) : The Human Relations Theory focuses on management as a web of interpersonal relations.Reason (R) : Richard Franke and James Kaul concluded that the human relations were not the reason behind worker’s efficiency.Ans : (C)

95. Assertion (A) : The Government has no power to regulate, through an appropriate legislation, the recruitment to civil services.Reason (R) : The Constitution provides for the establishment of an impartial and independent Union Public Service Commission for recruiting civil servants in India.Ans : (D)

96. Assertion (A) : In the open model, what is good for the individual is also good for the society.Reason (R) : Open model of organization, like the closed model, is an ideal type.Ans : (B)

97. Assertion (A) : Individuals and organizations over invest in information.Reason (R) : Information intensive decisions have a greater organizational acceptance and smoother implementation.Ans : (A)

98. Assertion (A) : The first principle of democratic governance is that the rulers should be accountable to the people.Reason (R) : Transparency and accountability in governance depend on citizens easy access to reliable information and data.Ans : (B)

99. Assertion (A) : The ‘disadvantaged’ states perceive that they have been punished for the better performance, whereas the ‘laggard’ states continue to be rewarded.Reason (R) : The Finance Commission has the constitutional responsibility to provide succour to the underdeveloped States.Ans : (B)

100. Assertion (A) : The bureaucracy and the political leadership can achieve the goals by working closely and harmoniously.Reason (R) : If political leadership represents the input functions of a political system, the output function

Page 72: Commerce&Audit

is performed by the administration.Ans : (A)